Buy BOOKS at Discounted Price

Probability

Class 11th Mathematics RD Sharma Solution
Exercise 33.1
  1. A coin is tossed once. Write its sample space.
  2. If a coin is tossed two times, describe the sample space associated to this…
  3. If a coin is tossed three times (or three coins are tossed together), then…
  4. Write the sample space for the experiment of tossing a coin four times.…
  5. Two dice are thrown. Describe the sample space of this experiment.…
  6. What is the total number of elementary events associated to the random…
  7. A coin is tossed and then a die is thrown. Describe the sample space for this…
  8. A coin is tossed and then a die is rolled only in case a head is shown on the…
  9. A coin is tossed twice. If the second throw results I a tail, a die is thrown.…
  10. An experiment consists of tossing a coin and then tossing it second time if…
  11. A coin is tossed. If it shows tail, we draw a ball from a box which contains 2…
  12. A coin is tossed repeatedly until a tail comes up for the first time. Write…
  13. A box contains 1 red and 3 black balls. Two balls are drawn at random in…
  14. A pair of dice is rolled. If the outcome is a doublet, a coin is tossed.…
  15. A coin is tossed twice. If the second draw results in a head, a die is rolled.…
  16. A bag contains 4 identical red balls and 3 identical black balls. The…
  17. In a random sampling three items are selected from a lot. Each item is tested…
  18. An experiment consists of boy-girl composition of families with 2 children.…
  19. There are three coloured dice of red, white and black colour. These dice are…
  20. 2 boys and 2 girls are in room P and 1 boy 3 girls are in room Q. Write the…
  21. A bag contains one white and one red ball. A ball is drawn from the bag. If…
  22. A box contains 1 white and 3 identical black balls. Two balls are drawn at…
  23. An experiment consists of rolling a die and then tossing a coin once it number…
  24. A die is thrown repeatedly until a six comes up. What is the sample space for…
Exercise 33.2
  1. A coin is tossed. Find the total number of elementary events and also the total…
  2. List all events associated with the random experiment of tossing of two coins.…
  3. Three coins are tossed once. Describe the following events associated with this…
  4. In a single throw of a die describe the following events: (i) A = Getting a…
  5. Three coins are tossed. Describe (i) two events A and B which are mutually…
  6. A die is thrown twice. Each time the number appearing on it is recorded.…
  7. Two dice are thrown. The events A, B, C, D, E and F are describes as follows. A…
  8. The numbers 1, 2, 3 and 4 are written separately on four slips of paper. The…
  9. A card is picked up from a deck of 52 playing cards. (i) What is the sample…
Exercise 33.3
  1. Which of the following cannot be valid assignment of probability for elementary…
  2. A die is thrown. Find the probability of getting: (i) a prime number (ii) 2 or…
  3. In a simultaneous throw of a pair of dice, find the probability of getting: (i)…
  4. In a single throw of three dice, find the probability of getting a total of 17…
  5. Three coins are tossed together. Find the probability of getting: (i) exactly…
  6. What is the probability that an ordinary year has 53 Sundays?
  7. What is the probability that a leap year has 53 Sundays and 53 Mondays?…
  8. A bag contains 8 red and 5 white balls. Three balls are drawn at random. Find…
  9. In a single throw of three dice, find the probability of getting the same…
  10. Two unbiased dice are thrown. Find the probability that the total of the…
  11. A card is drawn at random from a pack of 52 cards. Find the probability that…
  12. In shutting a pack of 52 playing cards, four are accidently dropped; find the…
  13. From a deck of 52 cards, four cards are drawn simultaneously, find the chance…
  14. Tickets numbered from 1 to 20 are mixed up together and then a ticket is drawn…
  15. A bag contains 6 red, 4 white and 8 blue balls. If three balls are drawn at…
  16. A bag contains 7 white, 5 black and 4 red balls. If two balls are drawn at…
  17. A bag contains 6 red, 4 white and 8 blue balls. If three balls are drawn at…
  18. Five cards are drawn from a pack of 52 cards. What is the chance that these 5…
  19. The face cards are removed from a full pack. Out of the remaining 40 cards, 4…
  20. There are four men and six women on the city councils. If one council member…
  21. A box contains 100bulbs, 20 of which are defective. 10 bulbs are selected for…
  22. Find the probability that in a random arrangement of the letters of the word…
  23. The letters of the word ‘CLIFTON’ are placed at random in a row. What is the…
  24. The letters of the word “FORTUNATES’ are arranged at random in a row. What is…
  25. A committee of two persons is selected from two men and two women. What is the…
  26. If odds in favour of an event be 2: 3, find the probability of occurrence of…
  27. If odds against an events be 7: 9, find the probability of non-occurrence of…
  28. Two balls are drawn at random from a bag containing 2 white, 3 red, 5 green…
  29. Two unbiased dice are thrown. Find the probability that: (i) neither a doublet…
  30. A bag contains 8 red, 3 white and 9 blue balls. If three balls are drawn at…
  31. A bag contains 5 red, 6 white and 7 black balls. Two balls are drawn at…
  32. If a letter is chosen at random from the English alphabet, find the…
  33. In a lottery, a person chooses six different numbers at random from 1 to 20,…
  34. 20 cards are numbered from 1 to 20. One card is drawn at random. What is the…
  35. Two dice are thrown. Find the odds in favour of getting the sum (i) 4 (b) 5…
  36. What are the odds in favour of getting a spade if a card is drawn from a…
  37. A box contains 10 red marbles, 20 blue marbles and 30 green marbles. 5 marbles…
  38. A box contains 6 red marbles numbered 1 through 6 and 4 white marbles numbered…
  39. A class consists of 10 boys and 8 girls. Thee students are selected at random.…
  40. Five cards are drawn from a well-shuffled pack of 52 cards. Find the…
  41. A bag contains tickets numbered from 1 to 20. Two tickets are drawn. Find the…
  42. An urn contains 7 white, 5 black and 3 red balls. Two balls are drawn at…
  43. A and B throw a pair of dice. If A throws 9, find B’s chance of throwing a…
  44. In a hand at Whist, what is the probability that four kings are held by a…
  45. Find the probability that in a random arrangement of the letters of the word…
Exercise 33.4
  1. If A and B be mutually exclusive events associated with a random experiment…
  2. A and B are two events such that P(A) = 0.54, P(B) = 0.69 and P(A ∩ B) =0.35.…
  3. Fill in the blanks in the following table :
  4. If A and B are two events associated with a random experiment such that P(A) =…
  5. If A and B are two events associated with a random experiment such that P(A) =…
  6. If A and B are two events associated with a random experiment such that P(A ∪…
  7. Given two mutually exclusive events A and B such that P(A) = 1/2 and P(B) =…
  8. There are three events A, B, C one of which must and only one can happen, the…
  9. One of the two events must happen. Given that the chance of one is two-third of…
  10. A card is drawn at random from a well-shuffled deck of 52 cards. Find the…
  11. In a single throw of two dice, find the probability that neither a doublet nor…
  12. A natural number is chosen at random from amongst first 500. What is the…
  13. A die is thrown twice. What is the probability that at least one of the two…
  14. A card is drawn from a deck of 52 cards. Find the probability of getting an…
  15. The probability that a student will pass the final examination in both English…
  16. One number is chosen from numbers 1 to 100. Find the probability that it is…
  17. From a well shuffled deck of 52 cards, 4 cards are drawn at random. What is…
  18. 100 student appeared for two examinations. 60 passed the first, 50 passed the…
  19. A box contains 10 white, 6 red and 10 black balls. A ball is drawn at random…
  20. In a race, the odds in favour of horses A,B,C,D are 1:3, 1:4, 1:5 and 1:6…
  21. The probability that a person will travel by plane is 3/5 and that he will…
  22. Two cards are drawn from a well shuffled pack of 52 cards. Find the…
  23. In an entrance test that is graded on the basis of two examinations, the…
  24. A box contains 30 bolts and 40 nuts. Half of the bolts and half of the nuts…
  25. An integer is chosen at random from first 200 positive integers. Find the…
  26. Find the probability of getting 2 or 3 tails when a coin is tossed four times.…
  27. Suppose an integer from 1 through 1000 is chosen at random, fins the…
  28. In a large metropolitan area, the probabilities are 0.87, 0.36, 0.30 that a…
  29. If A and B are mutually exclusive events such that P(A) = 0.35 and P(B) =…
  30. A sample space consists of 9 elementary event E1, E2, E3, ……E8, E9 whose…
Very Short Answer
  1. Three numbers are chosen at random from numbers 1 to 30. Write the probability that the…
  2. n (3) persons are sitting in a row. Two of them are selected. Write the probability…
  3. A single letter is selected at random from the word ‘PROBABILITY’. What is the…
  4. What is the probability that a leap year will have 53 Fridays or 53 Saturday?…
  5. Three dice are thrown simultaneously. What is the probability of getting 15 as the sum?…
  6. If the letters of the word ‘MISSISSIPPI’ are written down at random in a row, what is…
  7. What is the probability that the 13th days of a randomly chosen month is Friday?…
  8. Three of the six vertices of a regular hexagon are chosen at random. What is the…
  9. If E and E2 are independent events, write the value of p (e_1 union e_2) integrate…
  10. If A and B are two independent events such that p (a integrate erdection b) = 1/6 p…
Mcq
  1. One card is drawn from a pack of 52 cards. The probability that it is the card of a…
  2. Two dice are thrown together. The probability that at least one will show its digit…
  3. Two dice are thrown simultaneously. The probability of obtaining a total score of 5…
  4. Two dice are thrown simultaneously. The probability of obtaining total score of seven…
  5. The probability of getting a total of 10 in a single throw of two dice isA. 1/9 B. 1/12…
  6. A card is drawn at random from a pack of 100 cards numbered 1 to 100. The probability…
  7. A bag contains 3 red, 4 white and 5 blue balls. All balls are different. Two balls are…
  8. Two dice are thrown together. The probability that neither they show equal digits nor…
  9. Four persons are selected at random out of 3 men, 2 women and 4 children. The…
  10. The probabilities of happening of two events. A and B are 0.25 and 0.50 respectively.…
  11. A die is rolled, then the probability that a number 1 or 6 may appear isA. 2/3 B. 5/6…
  12. Six boys and six girls sit in a row randomly. The probability that all girls sit…
  13. The probabilities of three mutually exclusive events A, B and C are given by 2/3, 1/4…
  14. If (1-3p)/2 , (1+4p)/3 , (1+p)/6 are the probabilities of three mutually exclusive and…
  15. A pack of cards contains 4 aces, 4 kings, 4 queens and 4 jacks. Two cards are drawn at…
  16. If three dice are thrown simultaneously, then the probability of getting a score of 5…
  17. One of the two events must occur. If the chance of one is 2/3 of the other, then odds…
  18. The probability that a leap year will have 53 Fridays or 53 Saturdays isA. 2/7 B. 3/7…
  19. A person write 4 letters and addresses 4 envelopes. If the letters are placed in the…
  20. A and B are two events such that P(A) = 0.25 and P(B) = 0.50. The probability of both…
  21. If the probability of A to fail in an examination is 1/5 and that of B is 3/10 . Then,…
  22. A box contains 10 good articles and 6 defective articles. One item is drawn at random.…
  23. Three integers are chosen at random from the first 20 integers. The probability that…
  24. Out of 30 consecutive integers, 2 are chosen at random. The probability that their sum…
  25. A bag contains 5 black balls, 4 white balls and 3 red balls. If a ball is selected…
  26. Two dice are thrown simultaneously. The probability of getting a pair of sixes isA.…
  27. An urn contains 9 balls two of which are red, three blue and four black. Three balls…
  28. Five persons entered the lift cabin on the ground floor of an 8 floored house. Suppose…
  29. A box contains 10 good articles and 6 with defects. One item is drawn at random. The…
  30. A box contains 6 nails and 10 nuts. Half of the nails and half of the nuts are rusted.…
  31. If S is the sample space and p (a) = 1/3 p (b) s = a union b , where A and B are two…
  32. One mapping is selected at random from all the mappings of the set A = {1, 2, 3, …..,…
  33. If A, B, C are three mutually exclusive and exhaustive events of an experiment such…
  34. If A and B are mutually exclusive events thenA. p (a) less than equal to p (bar b) B.…
  35. If P (A ∩ B) = P (A ∪ B) for any two events A and B, thenA. P (A) = P (B) B. P (A) P…
  36. Three numbers are chosen from 1 to 20. The probability that they are not consecutive…
  37. 6 boys and 6 girls sit in a row a row at random. The probability that all the girls…
  38. Without repetition of the numbers, four digit numbers are formed with the numbers 0,…
  39. If the probability for A to fail in an examination is 0.2 and that for B is 0.3, then…
  40. Three digit numbers are formed using the digits 0, 2, 4, 6, 8. A number is chosen at…

Exercise 33.1
Question 1.

A coin is tossed once. Write its sample space.


Answer:

Given: A coin is tossed once.


To Find: Write its sample space?


Explanation: Here, the coin is tossed only once,


Then, there are two probability either Head(H) or Tail(T)


So, Sample will be


S = {H, T}


Where, H denotes Head and T denotes Tail


Hence, The sample is {H, T}



Question 2.

If a coin is tossed two times, describe the sample space associated to this experiment.


Answer:

Given: If Coin is tossed twice times.


To Find: Write the sample space associated to this experiment.


Explanation: Here, two coins are tossed, that means two probability will occur at same time


So, The sample space will be


S={HT, TH, HH, TT}


Hence, Sample space is {HT, HH, TT, TH}



Question 3.

If a coin is tossed three times (or three coins are tossed together), then describe the sample space for this experiment.


Answer:

Given: If a coin is tossed three times .


To Find: Write the sample space for the given experiment.


Explanation: Here, the coins is tossed three time, then the no. of samples


23=8


So, The sample space will be


S={HHH, TTT, HHT, HTH, THH, HTT, THT, TTH}


Hence, The sample space is {HHH, TTT, HHT, HTH, THH, HTT, THT, TTH}



Question 4.

Write the sample space for the experiment of tossing a coin four times.


Answer:

Given: A coin is tossed four times.


To Find: Write the sample space for the given experiment.


Explanation: Here, The coins is tossed four time, then the no. of samples


24=16


So, The sample space will be


S={HHHH, TTTT, HHHT, HHTH, HTHH, THHH, HHTT, HTTH, HTHT, THHT, THTH, TTHH, HTTT, THTT, TTHT, TTTH}


Hence, The sample space is {HHHH, TTTT, HHHT, HHTH, HTHH, THHH, HHTT, HTTH, HTHT, THHT, THTH, TTHH, HTTT, THTT, TTHT, TTTH}



Question 5.

Two dice are thrown. Describe the sample space of this experiment.


Answer:

Given: Two dice are thrown.


To Find: Write the sample space for the given experiment.


Explanation: We know there are 6 faces on a dice. Contains (1, 2, 3, 4, 5, 6).


But, Here two dice are thrown, then we have two faces of dice (one of each)


So, The total sample space will be 62 = 36


Now, the sample space is:


S={(1, 1), (1, 2), (1, 3), (1, 4), (1, 5), (1, 6), (2, 1), (2, 2), (2, 3), (2, 4), (2, 5), (2, 6), (3, 1)(3, 2), (3, 3), (3, 4), (3, 5), (3, 6), (4, 1), (4, 2), (4, 3), (4, 4), (4, 5), (4, 6), (5, 1), (5, 2), (5, 3), 5, 4), (5, 6), (5, 5), (6, 1), (6, 2), (6, 3), (6, 4), (6, 5), (6, 6)}



Question 6.

What is the total number of elementary events associated to the random experiment of throwing three dice together?


Answer:

Given: Three dice is rolled together.


To Find: What is the total number of elementary events.


Explanation: Here, three dice are thrown together,


And, There are 6 faces on die,


So, The total number of elementary event on throwing three dice are


6×6×6=216


Hence, The total number is 216



Question 7.

A coin is tossed and then a die is thrown. Describe the sample space for this experiment.


Answer:

Given: A coin is tossed and a die is thrown.


To Find: Write the sample space for the given experiment.


Explanation: Here, The coin is tossed and die is thrown.


We know, when coin is tossed there will be 2 events either Head or Tail.


And, when die is thrown then there will be 6 faces (1, 2, 3, 4, 5, 6)


SO, The total number of Sample space together is 2×6 = 12


S={(H, 1), (H, 2), (H, 3), (H, 4), (H, 5), (H, 6), (T, 1), (T, 2), (T, 3), (T, 4), (T, 5), (T, 6)}


Hence, Sample space are {(H, 1), (H, 2), (H, 3), (H, 4), (H, 5), (H, 6), (T, 1), (T, 2), (T, 3), (T, 4), (T, 5), (T, 6)}



Question 8.

A coin is tossed and then a die is rolled only in case a head is shown on the coin. Describe the sample space for this experiment.


Answer:

Given: A coin is tossed and the a die is rolled.


To Find: Write the sample space for the given experiment.


Explanation: Here, we have a coin and a die,


We know, when coin is tossed there will be 2 event Head and tail,


According to question, If Head occurs on coin then Die will rolled out otherwise not.


So, the sample spaces are :


S={(T, (H, 1)(H, 2), (H, 3), (H, 4), (H, 5), (H, 6)}


Hence, Sample space is (T, (H, 1)(H, 2), (H, 3), (H, 4), (H, 5), (H, 6)



Question 9.

A coin is tossed twice. If the second throw results I a tail, a die is thrown. Describe the sample space for this experiment.


Answer:

Given: A coin is tossed twice. If the second throw results I a tail, A die is thrown.


To Find: Write the sample space for the given experiment.


Explanation: When a coin tossed twice, Then sample spaces for only coin will be: {HH, TT, HT , TH}


Now, According to question , when we get Tail in second throw, then a dice is thrown.


So, The total number of elementary events are 2+(2×6)=14


And sample space will be


S={HH, TH, (HT, 1), (HT, 2), (HT, 3), (HT, 4), (HT, 5), (HT, 6), (TT, 1), (TT, 2), (TT, 3), (TT, 4), (TT, 5), (TT, 6)}


Hence, this is the sample space for given experiment.



Question 10.

An experiment consists of tossing a coin and then tossing it second time if head occurs. If a tail occurs on the first toss, then a die is tossed once. Find the sample space.


Answer:

Given: A coin is tossed and a die is rolled.


To Find: Write the sample space for the given experiment.


Explanation: In the given experiment, coin is tossed and if the outcome is tail then, die will be rolled.


The possible outcome for coin is 2 = {H, T}


And, The possible outcome for die is 6 = {1, 2, 3, 4, 5, 6}


If the outcome for the coin is tail then sample space is S1={(T, 1)(T, 2)(T, 3)(T, 4)(T, 5)(T, 6)}


If the outcome is head then the sample space is S2={(H, H)(H, T)}


So, The required outcome sample space is S=S1S2


S={(T, 1)(T, 2)(T, 3)(T, 4)(T, 5)(T, 6)(H, H)(H, T)}


Hence, The sample space for the given experiment.



Question 11.

A coin is tossed. If it shows tail, we draw a ball from a box which contains 2 red 3 black balls; it shows head, we throw a die. Find the sample space of this experiment.


Answer:

Given: A coin is tossed and there is box which contain 2 red and 3 black balls.


To Find: Write the sample space for the given experiment.


Explanation: when coin is tossed , there are 2 outcome {H, T}


According to question, If tail turned up, the a ball is drawn from a box.


So, Sample for This experiment S1 = {(T, R1)(T, R2)(T, B1)(T, B2)(T, B3)}


Now, If Head is turned up, then die is rolled


So, Sample space for this experiment S2={(H, 1)(H, 2)(H, 3)(H, 4)(H, 5)(H, 6)}


The required sample space will be S = S1 S2


So, S={(T, R1), (T, R2), (T, B1), (T, B2), (T, B3), (H, 1), (H, 2), (H, 3), (H, 4), (H, 5), (H, 6)}


Hence, S is the elementary events associated with the given experiment.



Question 12.

A coin is tossed repeatedly until a tail comes up for the first time. Write the sample space for this experiment.


Answer:

Given: A coin is tossed repeatedly until comes up fro the first time.


To Find: Write the sample space for the given experiment.


Explanation: In the given Experiment, a coin is tossed and if the outcome is tail the experiment is over ,


And, if the outcome is Head then the coin is tossed again.


In the second toss also if the outcome is tail then experiment is over, otherwise coin is tossed again.


This process continues indefinitely


SO, The sample space for this experiment is


S={T, HT, HHT, HHHT, HHHHT…}


Hence, S is th sample space for the given experiment.



Question 13.

A box contains 1 red and 3 black balls. Two balls are drawn at random in succession without replacement. Write the sample space for this experiment.


Answer:

Given: A box contains 1 red and 3 black balls.


To Find: Write the sample space for the given experiment.


Explanation: We have 1 red and 3 black balls in a box.


Let Assume Red = R


Let Assume Blue = B


Since, two balls are drawn at random without replacement,


So, The sample spaces for this experiment is:


S={(R, B1), (R, B2), (R, B3)(B1, B2)(B1, B3)(B1, R)(B2, R)(B2, B1)(B2, B3)(B3, R)(B3, B1)(B3, B2)}


Hence, S is the sample spaces for given experiment.



Question 14.

A pair of dice is rolled. If the outcome is a doublet, a coin is tossed. Determine the total number of elementary events associated to this experiment.


Answer:

Given: A pair of dice is rolled , a coin is tossed.


To Find: Write the sample space for the given experiment.


Explanation: A pair of dice is rolled,


Then, No. of elementary events are 62=36


Now, If outcomes is doublet means (1, 1)(2, 2)(3, 3)(4, 4)(5, 5)(6, 6), then a coin is tossed.


If coin is tossed then no. of sample spaces is 2


So, The total no. of elementary events including doublet = 6×2=12


Thus, The Total number of elementary events are 30+12 =42


Hence, 42 events will occur for this experiments.



Question 15.

A coin is tossed twice. If the second draw results in a head, a die is rolled. Write the sample space for this experiment.


Answer:

Given: There is a coin which tossed twice.


To Find: Write the sample space for the given experiment.


Explanation: A coin is tossed twice , So the outcomes are


S1={HH, HT, TH, TT}


Now, If the second drawn result is head , the a die is rolled then the elementary events is


S2={(HH, 1), (HH, 2)(HH, 3), (HH, 4)(HH, 5), (HH, 6), (HH, 1), (TH, 2)(TH, 3), (TH, 4)(TH, 5), (TH, 6)}


Thus, The total sample space for the experiment is S=S1 S2


S={(HH), (HT), (TH), (TT)(HH, 1), (HH, 2)(HH, 3), (HH, 4)(HH, 5), (HH, 6), (HH, 1), (TH, 2)(TH, 3), (TH, 4)(TH, 5), (TH, 6)}


Hence, S is the sample space for given experiment.



Question 16.

A bag contains 4 identical red balls and 3 identical black balls. The experiment consists of drawing one ball, then putting it into the bag and again drawing a ball. What are the possible outcomes of the experiment?


Answer:

Given: A bag contains 4 identical red balls and 3 identical black balls.


To Find: Write the sample space for the given experiment.


Explanation: A bag contains 4 red balls and 3 black balls.


Let us Assume Red = R


Let us Assume Black = B


Now, A ball is drawn in first attempt, So elementary events is


S1={R, B}


And, The bag will put into the bag and draw again, then elementary events are


S2={R, B}


Thus, The total sample space associated is S=S1S2


S={RR, RB, BR, BB}


Hence, S is the sample space for given experiment.



Question 17.

In a random sampling three items are selected from a lot. Each item is tested and classified as defective (D) or non-defective (N). Write the sample space of this experiment.


Answer:

Given: There are three sampling item.


To Find: Write the sample space for the given experiment.


Explanation: In the random sampling , three items are selected so it could be


(a) All defective (D)


(b) All non-defective(N)


(c) Combination of both defective and non defective


We have 2 category (N and D) and we have three condition


So, The number of sample is 23 = 8


Sample space associated with this experiment is


S={DDD, NDN, DND, DNN, NDD, DDN, NND, NNN)


Hence, S is the sample space for given experiment



Question 18.

An experiment consists of boy-girl composition of families with 2 children.

(i) What is the sample space if we are interested in knowing whether it is boy or girl in the order of their births?

(ii) What is the sample space if we are interested in the number of boys in a family?


Answer:

(i) To Find: Write the sample space for the given experiment.


Explanation: Here, The family has 2 children


Let us Assume Boy = B


Let us Assume Girl = G


So, The number of sample spaces for 2 children is 22=4


Sample space are, S={(B1, B2), (G1, G2), (G1, B2), (B1, G2)}


Where , number 1 and 2 are represent elder and younger.


Hence, S is the sample space for given experiment.


(ii) Explanation: Here, The family has 2 children, So the possibility of boys in a family is:


(a) No boys only girl


(b) One boy and one girl


(c) Two boys only


So, The Sample space for the given condition is:


S={0, 1, 2}


Hence, S is the sample spaces for the given experiment.



Question 19.

There are three coloured dice of red, white and black colour. These dice are placed in a bag. One die is drawn at random from the bag and rolled, its colour and the number on its uppermost face is noted. Describe the sample space for this experiment.


Answer:

Given: There are three colored dice of red , white and black color. These dice are placed in bag.


To Find: Write the sample space for the given experiment.


Explanation: A dice has 6 faces containing numbers (1, 2, 3, 4, 5, 6)


Let us Assume Red = R


Let us Assume White = W


Let us Assume Black = B


According to question, when Dice is selected , then rolled.


Firstly, selected Red Dice then, Possible samples are:


SR={(R, 1), (R, 2)(R, 3), (R, 4), (R, 5), (R, 6)}


Then, White Dice will be selected , So the sample spaces are:


Sw={(W, 1), (W, 2), (W, 3), (W, 4), (W, 5), (W, 6)}


Lastly, Black Dice will be selected and rolled , So the sample spaces for Black die


SB= {(B, 1)(B, 2)(B, 3)(B, 4)(B, 5), (B, 6)}


Thus, The total sample for the given experiment is


S=SR SW SB



Hence, B is the sample space for the given experiment.



Question 20.

2 boys and 2 girls are in room P and 1 boy 3 girls are in room Q. Write the sample space for the experiment in which a room is selected and then a person.


Answer:

Given: 2 boys and 2 girls are in room P and 1 boy 3 girls are in room Q.


To Find: Write the sample space for the given experiment.


Explanation: Let us denote two boys and girls in room P as B1 , B2 and G1 , G2 respectively


Let us denote 1 boy and 3 girls in room Q as B3 and G3, G4, G5 respectively.


According to the question,


Sample spaces for room Sp are


Sp={(P, B1), (P, B2), (P, G1), (P, G2)}


And, Sample spaces for room Sq are


Sq={(Q, B3), (Q, G1), (Q, G2), (Q, G3)}


Now, The total sample spaces for the given experiment:


S=


S={(P, B1), (P, B2), (P, G1), (P, G2), (Q, B3), (Q, G1), (Q, G2), (Q, G3)}


Hence, S is the sample space for the given experiment



Question 21.

A bag contains one white and one red ball. A ball is drawn from the bag. If the ball drawn is white it is replaced in the bag and again a ball is drawn. Otherwise, a die is tossed. Write the sample space for this experiment.


Answer:

Given: A bag contains one white and one red ball.


To Find: Write the sample space for the given experiment.


Explanation: Here, There are 1 white ball , 1 red ball in a bag


Let us denote White ball with W and Red Ball with W


According to question, when one ball is drawn, it may be a white or Red.


So, The sample space of drawing on white ball with replacement


Sw={(W, W), (W, R)}


Again, If red ball is drawn , a dice is rolled


So, The sample space for red ball with dice


SR={(R, 1), (R, 2), (R, 3), (R, 4), (R, 5), (R, 6)}


Thus, The required sample space for the experiment is:


S=SW SR


So, S= {(W, W), (W, R), (R, 1), (R, 2), (R, 3), (R, 4), (R, 5), (R, 6)}


Hence, This is the sample space for given experiment



Question 22.

A box contains 1 white and 3 identical black balls. Two balls are drawn at random in succession without replacement. Write the sample space for this experiment.


Answer:

Given: A box contain1 white and 3 identical black balls.


To Find: Write the sample space for the given experiment.


Explanation: Here, only 1 white ball and 3 Black identical Balls.


Let us denote the white ball with W and Black ball with B


When two balls are drawn at random in succession without replacement,


So, The sample space for this experiment is


S={WB, BW, BB}



Question 23.

An experiment consists of rolling a die and then tossing a coin once it number on the die is even. If the number on the die is odd, the coin is tossed twice. Write the sample space for this experiment.


Answer:

Given: A dice is rolled and then a coin is tossed once.


To Find: Write the sample space for the given experiment.


Explanation: A dice has 6 faces that are numbered from 1 to 6.


And, we know, 2, 4, 6 are even numbers and 1, 3, 5 are odd numbers.


Now, A coin has two face, Head(H) and Tail(T)


So, we can find the total sample spaces by combining both experiments as


S={(2, H), (2, T), (4, H), (4, T), (6, H), (6, T), (1, HH), (1, HT), (1, TH), (1, TT), (3, HH), (3, HT) , (3, TH), (3, TT), (5, HH), (5, HT), (5, TH), (5, TT)}


Hence, S is the required sample space for given experiment.



Question 24.

A die is thrown repeatedly until a six comes up. What is the sample space for this experiment.


Answer:

Given: A die is thrown repeatedly until a six comes up.


To Find: Sample space for the given experiment


Explanation: Here, The die is thrown repeatedly until a six comes up. It means


Since, 6 may come upon the first throw , second throw , third throw and so on until is obtained.


So, This process will continues indefinitely, then the sample space will be:


S={6, (1, 6), (2, 6), (3, 6), (4, 6), (5, 6), (1, 1, 6), (1, 2, 6), (1, 3, 6), (1, 4, 6), (1, 5, 6), (2, 1, 6), (2, 2, 6), (2, 3, 6), (2, 4, 6), (2, 5, 6)……..}


Hence, S is the sample space for given experiment.




Exercise 33.2
Question 1.

A coin is tossed. Find the total number of elementary events and also the total number of events associated with the random experiment.


Answer:

Given: A coin is tossed.


To Find: Find the total number of elementary events and a total number of events associated with the random experiment.


Explanation: When a coin is tossed, there will be two possible outcomes, Head(H) and Tail(T).


Since, the no. of elementary events is 2 {H, T}


But, we know, if there are n elements in a set, then the number of total element in its subset is 2n.


Therefore, the total number of the experiment is 4,


So, there are 4 subset of S = {H}, {T}, {H, T} and Փ


Hence, there are 4 total events in a given experiment.



Question 2.

List all events associated with the random experiment of tossing of two coins. How many of them are elementary events?


Answer:

Given: Two coins are tossed once.


To find: How many events are elementary events


Explanation: We know, when Two coins are tossed then the no. of possible outcomes are 22 = 4


So, The Sample spaces are {HH, HT, TT, TH}


Hence, there are total 4 events associated with the given experiment.



Question 3.

Three coins are tossed once. Describe the following events associated with this random experiment:

A = Getting three heads, B = Getting two heads and one tail, C = Getting three tails, D = Getting a head on the first coin.

(i) Which pairs of events are mutually exclusive?

(ii) Which events are elementary events?

(iii) Which events are compound events?


Answer:

Given: There are three coins tossed once.


To Find: Describe the events according to the subparts?


Explanation: when three coins are tossed, then the sample spaces are:


S = {HHH, HHT, HTH, HTT, THH, THT, TTH, TTT}


According to the question,


A = {HHH}


B = {HHT, HTH, THH}


C = {TTT}


D = {HHH, HHT, HTH, HTT}


Now,



Since, If the intersection of two sets are null or empty it means both the sets are Mutually Exclusive.


(i) Events A and B, Events A and C, Events B and C and events C and D are mutually exclusive.


(ii) Here, We know, If an event has only one sample point of a sample space, then it is called elementary events.


So, A and C are elementary events.


(iii) If There is an event that has more than one sample point of a sample space, it is called a compound event,


Since,


So, B and D are compound events.



Question 4.

In a single throw of a die describe the following events:

(i) A = Getting a number less than 7

(ii) B = Getting a number greater than 7

(iii) C = Getting a multiple of 3

(iv) D = Getting a number less than 4

(v) E = Getting an even number greater than 4.

(vi) F = Getting a number not less than 3.

Also, find A ∪ B, A ∩ B, B ∩ C, E ∩ F, D ∩ F and .


Answer:

Given: A dice is thrown once.


To Find: Find the given events, and also find the Also, find A ∪ B, A ∩ B, B ∩ C, E ∩ F, D ∩ F and


Explanation: In a single throw of a die, the possible events are:


S = {1, 2, 3, 4, 5, 6}


Now, According to the subparts of the question, we have certain events as:


(i) A = getting a number below 7


So, The sample spaces for A are:


A = {1, 2, 3, 4, 5, 6}


(ii) B = Getting a number greater than 7


So, the sample spaces for B are:


B = {Փ}


(iii) C = Getting multiple of 3


So, The Sample space of C is


C = {3, 6}


(iv) D = Getting a number less than 4


So, The sample space for D is


D = {1, 2, 3}


(v) E = Getting an even number greater than 4.


The sample space for E is


E = {6}


(vi) F = Getting a number not less than 3.


The sapmle space for F is


F = {3, 4, 5, 6}


Now,


A = {1, 2, 3, 4, 5, 6} and B = {Փ}



A = {1, 2, 3, 4, 5, 6} and B = {Փ}



B = {Փ} and C = {3, 6}



F = {3, 4, 5, 6}and E = {6}



E = {6} and D = {1, 2, 3}



And, For


S = {1, 2, 3, 4, 5, 6} and F = {3, 4, 5, 6}



Hence, These are the events for given ecperiment



Question 5.

Three coins are tossed. Describe

(i) two events A and B which are mutually exclusive.

(ii) three events A, B and C which are mutually exclusive and exhaustive.

(iii) two events A and B which are not mutually exclusive.

(iv) two events A and B which are mutually exclusive but not exhaustive.


Answer:

When three coins are tossed, then the sample space is


S = {HHH, HHT, HTH, HTT, THH, THT, TTH, TTT}


Now, The subparts are:


(i) The two events which are mutually exclusive are when,


A: getting no tails


B: getting no heads


Then, A = {HHH} and B = {TTT}


SO, The intersection of this set will be null.


Or, The sets are disjoint.


(ii) Three events which are mutually exclusive and exhaustive are:


A: getting no heads


B: getting exactly one head


C:getting at least two head


So, A = {TTT} B = {TTH, THT, HTT} and C = {HHH, HHT, HTH, THH}


Since,


(iii) The two events that are not mutually exclusive are:


A:getting three heads


B:getting at least 2 heads


So, A = {HHH} B = {HHH, HHT, HTH, THH}


Since


(iv) The two events which are mutually exclusive but not exhaustive are:


A:getting exactly one head


B: getting exactly one tail


So, A = {HTT, THT, TTH} and B = {HHT, HTH, THH}


It is because



Question 6.

A die is thrown twice. Each time the number appearing on it is recorded. Describe the following events:

(i) A = Both numbers are odd.

(ii) B = Both numbers are even

(iii) C = sum of the numbers is less than 6.

Also, find A ∪ B, A ∩ B, A ∪ C, A ∩ C. Which pairs of events are mutually exclusive.


Answer:

Given: A dice is thrown twice. And each time number appearing on it is recorded.


To Find: Describe the given events.


Explanation: when the dice is thrown twice then the number of sample spaces are 62 = 36


Now,


The possibility both odd numbers are:


A = {(1, 1), (1, 3), (1, 5), (3, 1), (3, 3), (3, 5), (5, 1), (5, 3), (5, 5)}


Since, Possibility of both even numbers are:


B = {(2, 2)(2, 4)(2, 6)(4, 2)(4, 4)(4, 6)(6, 2)(6, 4)(6, 6)}


And, Possible outcome of sum of the numbers is less than 6


C = {(1, 1)(1, 2)(1, 3)(1, 4)(2, 1)(2, 2)(2, 3)(3, 1)(3, 2)(4, 1)}


Therefore,


(AՍB) = {(1, 1), (1, 3), (1, 5), (3, 1), (3, 3), (3, 5), (5, 1), (5, 3), (5, 5) (2, 2)(2, 4)(2, 6)(4, 2)(4, 4)(4, 6)(6, 2)(6, 4)(6, 6)}


(AՌB) = {Փ}


(AUC) = {(1, 1), (1, 3), (1, 5), (3, 1), (3, 3), (3, 5), (5, 1), (5, 3), (5, 5) (1, 2)(1, 4)(2, 1)(2, 2)(2, 3)(3, 1)(3, 2)(4, 1)}


(AՌC) = {(1, 1)(1, 3)(3, 1)}


Hence, (AՌB) = Փ and (AՌC)≠Փ, A and B are mutually exclusive, but A and C are not.



Question 7.

Two dice are thrown. The events A, B, C, D, E and F are describes as follows.

A = Getting an even number on the first die.

B. Getting an odd number on the first die.

C = Getting at most 5 as sum of the numbers on the two dice.

D = Getting the sum of the numbers on the dice greater than 5 but less than 10.

E = Getting at least 10 as the sum of the numbers on the dice.

F = Getting an odd number on one of the dice.

(i) Describe the following events: A and B, B or C, B and C, A and E, A or F, A and F

(ii) State true or false:

(a) A and B are mutually exclusive.

(b) A and B are mutually exclusive and exhaustive events

(c) A and C are mutually exclusive events

(d) C and D are mutually exclusive and exhaustive events

(e) C, d and E are mutually exclusive and exhaustive events

(f) A’ and B’ are mutually exclusive events

(g) A, B, F are mutually exclusive and exhaustive events


Answer:

Given: Two dice are thrown.


To Find: Describe the given events.


Explanation: when two dice are thrown then the no. of possible outcomes are 62 = 36


Now, According to the question,


A = Getting an even number of the first die.


A = {(2, 1)(2, 2)(2, 3)(2, 4)(2, 5)(2, 6)(4, 1)(4, 2)(4, 3)(4, 4)(6, 1)(6, 2)(6, 3)(6, 4)(6, 5)(6, 6)}


B = Getting an odd number on the first dice


B = {(1, 1) (1, 2) (1, 3) (1, 4) (1, 5) (1, 6) (3, 1) (3, 2) (3, 3) (3, 4) (3, 5) (3, 6) (5, 1) (5, 2) (5, 3) (5, 4) (5, 5)(5, 6)}


C = Getting at most 5 as the sum of numbers on the two dices.


C = {(1, 1)(1, 2)(1, 3)(1, 4)(2, 1)(2, 2)(2, 3)(3, 1)(3, 2)(4, 1)}


D = Getting a sum greater than 5 but less than 10


D = {(1, 5)(1, 6) (2, 4)(2, 5) (2, 6)(3, 3)(3, 4)(3, 5)(3, 6)(4, 2)(4, 3)(4, 4)(4, 5)(5, 1) (5, 2)(5, 3) (5, 4)(6, 1) (6, 2)(6, 3)}


E = Getting at least 10 as the sum of numbers on the dices


{(4, 6)(5, 5)(5, 6)(6, 4)(6, 5)(6, 6)}


F = Getting an odd number on one of the dices


{(1, 2)(1, 4)(1, 6)(2, 1)(2, 3)(2, 5)(3, 2)(3, 4)(3, 6)(4, 1)(4, 3)(4, 5)(5, 2)(5, 4)(5, 6)(6, 1)(6, 3)(6, 5)


Now,


(i) A and B = AՌB = Փ


Since, There is no common events in Both A and B so the intersection is Null (Փ).


B or C = BUC


BUC = {(1, 1) (1, 2) (1, 3) (1, 4) (1, 5) (1, 6) (3, 1) (3, 2) (3, 3) (3, 4) (3, 5) (3, 6) (5, 1) (5, 2) (5, 3) (5, 4) (5, 5)(5, 6)(2, 1)(2, 2)(2, 3)(4, 1)}


B and C = BՌC


BՌC = {(1, 1), (1, 2)(1, 3)(1, 4)(3, 1)(3, 2)}


A and E = AՌE


AՌE = {(4, 6)(6, 4)(6, 5)(5, 6)(6, 6)}


A or F = AUF


AUF = {(2, 1)(2, 2)(2, 3)(2, 4)(2, 5)(2, 6)(4, 1)(4, 2)(4, 3)(4, 4)(6, 1)(6, 2)(6, 3)(6, 4)(6, 5)(6, 6)(1, 2)(1, 4)(1, 6)(3, 2)(3, 4)(3, 6)(4, 5)(5, 2)(5, 4)(5, 6)}


(ii)


(a) True, because AՌB = Փ


(b) True, because AՌB = Փ and AUB = S


(c) False, because AՌC = Փ


(d) False, because CՌD = Փ but CUD≠S


(e) True, because CՌDՌE = Փ and CUDUE = S


(f) True, because A’ՌB’ = Փ


(g) False, because AՌBՌF = Փ and AUBUF = S



Question 8.

The numbers 1, 2, 3 and 4 are written separately on four slips of paper. The slips are then put in a box and mixed thoroughly. A person draws two slips from the box, one after the other, without replacement. Describe the following events:

A = The number on the first slip is larger than the one on the second slip.

B = The number on the second slip is greater than 2

C = The sum of the numbers on the two slips is 6 or 7

D = The number on the second slips is twice that on the first slip.

Which pair (s) of events is (are) mutually exclusive?


Answer:

Given: There are 4 slips in the box and mixed thoroughly.


To Find: Describe the given events .


Explanation: Here, Four slips of paper 1, 2, 3, and 4 are put in a box.


If Two slips are drawn from it one after the other without replacement. Then


The sample space for the experiment is:


S = {(1, 2)(1, 3), (1, 4)(2, 1)(2, 3)(2, 4)(3, 1)(3, 2)(3, 4)(4, 1)(4, 2)(4, 3)}


(i) A = number on the first slip is larger than the one on the second slip,


So, The sample space for A is:


{(2, 1)(3, 1)(3, 2)(4, 1)(4, 2)(4, 3)}


(ii) B = number on the second slip is greater than 2


So, The sample space for B is


{(1, 3)(2, 3)(1, 4)(2, 4)(3, 4)(4, 3)}


(iii) C = sum of the numbers on the two slips in 6 or 7


The sample space for C is


{(2, 4)(3, 4)(4, 2)(4, 3)}


(iv) D = number on the second slip is two times the number on the first slip


The sample space if:


{(1, 2)(2, 4)}


Now,


We can see, AՌD = Փ


Therefore, A and D are mutually exclusive events


Hence, A and D are mutually exclusive events.



Question 9.

A card is picked up from a deck of 52 playing cards.

(i) What is the sample space of the experiment?

(ii) What is the event that the chosen card is a black faced card?


Answer:

Given: There is a deck of 52 playing card.


To Find: What is the sample space for the experiment and what is the event that the chosen card is black faced red.


Explanation: If a card is picked up from a deck of 52 playing cards, then the sample space of this experiment S is


(i) S = 52 cards in a deck


(ii) Let E is the event, and a black face card is chosen,


Then The possible outcomes in E are jack, queen, and king of clubs and the jack, queen and king of spades.


E = {J4, Q4, K4, Q*, KAM)


Hence, E = {J4, Q4, K4, Q*, KAM)




Exercise 33.3
Question 1.

Which of the following cannot be valid assignment of probability for elementary events or outcomes of sample space S = {w1, w2, w3, w4, w5, w6, w7}:


Answer:

for each event to be a valid assignment of probability, the probability of each event in sample space should be less than 1 and the sum of probability of all the events should be exactly equal to 1

(i) it is valid as each P(wi) (for i=1 to 7) lies between 0 to 1 and sum of P(w1) =1


(ii) it is valid as each P(wi) (for i=1 to 7) lies between 0 to 1 and sum of P(w1) =1


(iii) it is not valid as sum of P(wi)=2.8 which is greater than 1


(iv) it is not valid as which is greater than 1



Question 2.

A die is thrown. Find the probability of getting:

(i) a prime number

(ii) 2 or 4

(iii) a multiple of 2 or 3


Answer:

given: die is thrown

Therefore, the total number of outcomes is six


n(S)=6


formula:


(i) let E be the event of getting a prime number


E= {2,3,5}


n(E)=3




(ii) let E be the event of getting 2 or 4


E= {2,4}


n(E)=2




(iii) let E be the event of getting a multiple of 2 or 3


E= {2,3,4,6}


n(E)=4





Question 3.

In a simultaneous throw of a pair of dice, find the probability of getting:

(i) 8 as the sum

(ii) a doublet

(iii) a doublet of prime numbers

(iv) an even number on first

(v) a sum greater than 9

(vi) an even number on first

(vii) an even number on one and a multiple of 3 on the other

(viii) neither 9 nor 11 as the sum of the numbers on the faces

(ix) a sum less than 6

(x) a sum less than 7

(xi) a sum more than 7

(xii) neither a doublet nor a total of 10

(xiii) odd number on the first and 6 on the second

(xiv) a number greater than 4 on each die

(xv) a total of 9 or 11

(xvi) a total greater than 8


Answer:

given: a pair of dice have been thrown, so the number of elementary events in sample space is 62=36

formula:


therefore n(S)=36


(i) let E be the event that the sum 8 appears


E= {(2,6) (3,5) (4,4) (5,3) (6,2)}


n(E)=5




(ii) let E be the event of getting a doublet


E= {(1,1) (2,2) (3,3) (4,4) (5,5) (6,6)}


n(E)=6




(iii) let E be the event of getting a doublet of prime numbers


E= {((2,2) (3,3) (5,5)}


n(E)=3




(iv) let E be the event of getting a doublet of odd numbers


E= {(1,1) (3,3) (5,5)}


n(E)=3




(v) let E be the event of getting sum greater than 9


E= {(4,6) (5,5) (5,6) (6,4) (6,5) (6,6)}


n(E)=6




(vi) let E be the event of getting even on first die


E= {(2,1) (2,2) (2,3) (2,4) (2,5) (2,6) (4,1) (4,2) (4,3) (4,4) (4,5) (4,6) (6,1) (6,2) (6,3) (6,4) (6,5) (6,6)}


n(E)=18




(vii) let E be the event of getting even on one and multiple of three on other


E= {(2,3) (2,6) (4,3) (4,6) (6,3) (6,6) (3,2) (3,4) (3,6) (6,2) (6,4)}


n(E)=11




(viii) let E be the event of getting neither 9 or 11 as the sum


E= {(3,6) (4,5) (5,4) (5,6) (6,3) (6,5)}


n(E)=6




(ix) let E be the event of getting sum less than 6


E= {(1,1) (1,2) (1,3) (1,4) (2,1) (2,2) (2,3) (3,1) (3,2) (4,1)}


n(E)=10




(x) let E be the event of getting sum less than 7


E= {(1,1) (1,2) (1,3) (1,4) (1,5) (2,1) (2,2) (2,3) (2,4) (3,1) (3,2) (3,3) (4,1) (4,2) (5,1)}


n(E)=15




(xi) let E be the event of getting more than 7


E= {(2,6) (3,5) (3,6) (4,4) (4,5) (4,6) (5,3) (5,4) (5,5) (5,6) (6,2) (6,3) (6,4) (6,5) (6,6)}


n(E)=15




(xii) let E be the event of getting neither a doublet nor a total of 10


E’ be the event that either a doublet or a sum of ten appears


E’= {(1,1) (2,2) (3,3) (4,6) (5,5) (6,4) (6,6) (4,4)}


n(E’) =8




Therefore P(E)=1-P(E’)



(xiii) let E be the event of getting odd number on first and 6 on second


E= {(1,6) (5,6) (3,6)}


n(E)=3




(xiv) let E be the event of getting greater than 4 on each die


E= {(5,5) (5,6) (6,5) (6,6)}


n(E)=4




(xv) let E be the event of getting total of 9 or 11


E= {(3,6) (4,5) (5,4) (5,6) (6,3) (6,5)}


n(E)=6




(xvi) let E be the event of getting total greater than 8


E= {(3,6) (4,5) (4,6) (5,4) (5,5) (5,6) (6,3) (6,4) (6,5) (6,6)}


n(E)=10





Question 4.

In a single throw of three dice, find the probability of getting a total of 17 or 18


Answer:

given: three dices are thrown

formula:


Total number of possible outcomes are 63=216


Therefore n(S)=216


Let E be the event of getting total of 17 or 18


E= {(6,6,5) (6,5,6) (5,6,6) (6,6,6)}


n(E)=4





Question 5.

Three coins are tossed together. Find the probability of getting:

(i) exactly two heads

(ii) at least two heads

(iii) at least one head and one tail


Answer:

given: three coins are tossed together

formula:


Total number of possible outcomes are 23=8


(i) let E be the event of getting exactly two heads


E= {(H, H, T) (H, T, H) (T, H, H)}


n(E)=3




(ii) let E be the event of getting at least two heads


E= {(H, H, T) (H, T, H) (T, H, H) (H, H, H)}


n(E)=4




(iii) let E be the event of getting at least one head and one tail


E= {(H, T, T) (T, H, T) (T, T, H) (H, H, T) (H, T, H) (T, H, H)}


n(E)=6





Question 6.

What is the probability that an ordinary year has 53 Sundays?


Answer:

given: an ordinary year which includes 52 weeks and one day

formula:


so, we have to determine the probability of that one day being Sunday


Total number of possible outcomes are 7


Therefore n(S)=7


E= {M, T, W, T, F, S, SU}


n(E)=1





Question 7.

What is the probability that a leap year has 53 Sundays and 53 Mondays?


Answer:

given: a leap year which includes 52 weeks and two days

formula:


so, we have to determine the probability of that remaining two days are Sunday and Monday


S= {MT, TW, WT, TF, FS, SSu, SuM}


Therefore n(S)=7


E= {SuM}


n(E)=1





Question 8.

A bag contains 8 red and 5 white balls. Three balls are drawn at random. Find the probability that:

(i) All the three balls are white

(ii) All the three balls are red

(iii) One ball is red and two balls are white


Answer:

given: bag which contains 8 red and 5 white balls

formula:


total number of ways of drawing three balls at random is 13C3


therefore n(S)=286


(i) let E be the event of getting all white balls


E= {(W) (W) (W)}


n(E)= 5C3=10




(ii) let E be the event of getting all red balls


E= {(R) (R) (R)}


n(E)= 8C3=56




(iii) let E be the event of getting one red and two white balls


n(E)= 8C15C2=80





Question 9.

In a single throw of three dice, find the probability of getting the same number on all the three dice


Answer:

given: three dice are rolled

formula:


so, we have to determine the probability of getting the same number on all the three dice


total number of possible outcomes are 63=216


therefore n(S)=216


let E be the event of getting same number on all the three dice


E= {(1,1,1) (2,2,2) (3,3,3) (4,4,4) (5,5,5) (6,6,6)}


n(E)=6





Question 10.

Two unbiased dice are thrown. Find the probability that the total of the numbers on the dice is greater than 10


Answer:

given: three dice are rolled

formula:


so, we have to determine the probability of getting the sum of digits on dice greater than 10


total number of possible outcomes are 62=36


therefore n(S)=36


let E be the event of getting same number on all the three dice


E= {(5,6) (6,5) (6,6)}


n(E)=3





Question 11.

A card is drawn at random from a pack of 52 cards. Find the probability that the card drawn is ;

(i) a black king

(ii) either a black card or a king

(iii) black and a king

(iv) a jack, queen or a king

(v) neither an ace nor a king

(vi) spade or an ace

(vii) neither an ace nor a king

(viii) a diamond card

(ix) not a diamond card

(x) a black card

(xi) not an ace

(xii) not a black card


Answer:

given: pack of 52 cards

formula:


since a card is drawn from a pack of 52 cards, therefore number of elementary events in the sample space is


n(S)= 52C1 = 52


(i) let E be the event of drawing a black king


n(E)=2C1 =2 (there are two black kings one of spade and other of club)




(ii) let E be the event of drawing a black card or a king


n(E)=26C1+4C1-2C1=28


we are subtracting 2 from total because there are two black king which are already counted and to avoid the error of considering it twice




(iii) let E be the event of drawing a black card and a king


n(E)=2C1 =2 (there are two black kings one of spade and other of club)




(iv) let E be the event of drawing a jack, queen or king


n(E)=4C1+4C1+4C1=12




(v) let E be the event of drawing neither a heart nor a king


now consider E’ as the event that either a heart or king appears


n(E’) =6C1+4C1-1=16 (there is a heart king so it is deducted)




P(E)=1-P(E’)



(vi) let E be the event of drawing a spade or king


n(E)=13C1+4C1-1=16




(vii) let E be the event of drawing neither an ace nor a king


now consider E’ as the event that either an ace or king appears


n(E’) =4C1+4C1=8




P(E)=1-P(E’)



(viii) let E be the event of drawing a diamond card


n(E)=13C1=13




(ix) let E be the event of drawing not a diamond card


now consider E’ as the event that diamond card appears


n(E’) =13C1=13




P(E)=1-P(E’)



(x) let E be the event of drawing a black card


n(E)=26C1=26 (spades and clubs)




(xi) let E be the event of drawing not an ace


now consider E’ as the event that ace card appears


n(E’) =4C1=4




P(E)=1-P(E’)



(xii) let E be the event of not drawing a black card


n(E)=26C1=26 (red cards of hearts and diamonds)





Question 12.

In shutting a pack of 52 playing cards, four are accidently dropped; find the chance that the missing cards should be one from each suit


Answer:

given: pack of 52 cards from which 4 are dropped

formula:


we have to find the probability that the missing cards should be one from each suit


since from well shuffled pack of cards, 4 cards missed out total possible outcomes are


n(S)= 52C4=270725


let E be the event that four missing cards are from each suite


n(E)= 13C1×13C1×13C1×13C1=134





Question 13.

From a deck of 52 cards, four cards are drawn simultaneously, find the chance that they will be the four honors of the same suit


Answer:

given: deck of 52 cards

formula:


we have to find the probability that all the face cards of same suits are drawn


total possible outcomes are


n(S)=52C4


let E be the event that all the cards drawn are face cards of same suit


n(E)=4×4C4=4





Question 14.

Tickets numbered from 1 to 20 are mixed up together and then a ticket is drawn at random. What is the probability that the ticket has a number which is a multiple of 3 or 7?


Answer:

given: numbered tickets from 1 to 20

formula:


to find the probability of the ticket drawn having a number which is a multiple of 3 or 7


Since one ticket is drawn from a lot of mixed number, total possible outcomes are


n(S)=20C1=20


let E be the event of getting ticket which has number that is multiple of 3 or 7


E= {3,6,9,12,15,18,7,14}


n(E)=8





Question 15.

A bag contains 6 red, 4 white and 8 blue balls. If three balls are drawn at random, find the probability that one is red, one is white and one is blue


Answer:

given: 6 red, 4 white and 8 blue balls

formula:


three balls are drawn so, we have to find the probability that one is red, one is white and one is blue


total number of outcomes for drawing 3 balls are 18C3


Therefore n(S)= 18C3=816


Let E be the event that one red, one white and one blue ball is drawn


n(E)= 6C14C18C1=192





Question 16.

A bag contains 7 white, 5 black and 4 red balls. If two balls are drawn at random, find the probability that:

(i) both the balls are white

(ii) one ball is black and the other red

(iii) both the balls are of the same colour


Answer:

given: bag which contains 4 red, 5 black and 7 white balls

formula:


two balls are drawn at random, therefore


total possible outcomes are 16C2


therefore n(S)=120


(i) let E be the event of getting both white balls


E= {(W) (W)}


n(E)= 7C2=21




(ii) let E be the event of getting one black and one red ball


E= {(B) (R)}


n(E)= 5C14C1=20




(iii) let E be the event of getting both balls of same colour


E= {(B) (B)} or {(W) (W)} or {(R) (R)}


n(E)= 7C2+5C2+4C2=37





Question 17.

A bag contains 6 red, 4 white and 8 blue balls. If three balls are drawn at random, find the probability that:

(i) one is red and two are white

(ii) two are blue and one is red

(iii) one is red


Answer:

given: bag which contains 6 red, 8 blue and 4 white balls

formula:


two balls are drawn at random, therefore


total possible outcomes are 18C3


therefore n(S)=816


(i) let E be the event of getting one red and two white balls


E= {(W) (W) (R)}


n(E)= 6C14C2=36




(ii) let E be the event of getting two blue and one red


E= {(B) (B) (R)}


n(E)= 8C26C1=168




(iii) let E be the event that one of the balls must be red


E= {(R) (B) (B)} or {(R) (W) (W)} or {(R) (B) (W)}


n(E)= 6C14C18C1+6C14C2+6C18C2=396





Question 18.

Five cards are drawn from a pack of 52 cards. What is the chance that these 5 will contain:

(i) just one ace

(ii) at least one ace?


Answer:

given: pack of 52 playing cards

formula:


five cards are drawn at random, therefore


total possible outcomes are 52C5


therefore n(S)=2598960


(i) let E be the event that exactly only one ace is present


n(E)= 4C148C4=778320




(ii) let E be the event that at least one ace is present


E= {1 or 2 or 3 or 4 ace(s)}


n(E)= 4C148C4+4C248C3+4C348C2+4C448C1=886656





Question 19.

The face cards are removed from a full pack. Out of the remaining 40 cards, 4 are drawn at random. What is the probability that they belong to different suits?


Answer:

given: pack of 52 cards from which face cards are removed

formula:


four cards are drawn from the remaining 40 cards, so we have to find the probability that all of them belong to different suit


total possible outcomes of drawing four cards are 40C4


therefore n(S)= 40C4


let E be the event that 4 cards belong to different suit


n(E)= 10C110C110C110C1=10000





Question 20.

There are four men and six women on the city councils. If one council member is selected for a committee at random, how likely is that it is a women?


Answer:

given: four men and six women

formula:


from the city council one person is selected as a council member so, we have to find the probability that it is a woman


total possible outcomes of selecting a person is 10C1


therefore n(S)= 10C1


let E be the event that it is a woman


n(E)= 6C1=6





Question 21.

A box contains 100bulbs, 20 of which are defective. 10 bulbs are selected for inspection. Find the probability that:

(i) all 10 are defective

(ii) all 10 are good

(iii) at least one is defective

(iv) none is defective


Answer:

given: box with 100 bulbs of which, 20 are defective

formula:


ten bulbs are drawn at random for inspection, therefore


total possible outcomes are 100C10


therefore n(S)= 100C10


(i) let E be the event that all ten bulbs are defective


n(E)= 20C10




(ii) let E be the event that all ten good bulbs are selected


n(E)= 80C10




(iii) let E be the event that at least one bulb is defective


E= {1,2,3,4,5,6,7,8,9,10} where 1,2,3,4,5,6,7,8,9,10 are the number of defective bulbs


Let E’ be the event that none of the bulb is defective


n(E’) = 80C10




Therefore,


P(E)=1-P(E’)




(iv) let E be the event that none of the selected bulb is defective


n(E)= 80C10





Question 22.

Find the probability that in a random arrangement of the letters of the word ‘SOCIAL’ vowels come together


Answer:

given: word “SOCIAL”

formula:


In the random arrangement of the alphabets of word “SOCIAL” we have to find the probability that vowels come together


total possible outcomes of arranging the alphabets are 6!


therefore n(S)=6!


let E be the event that vowels come together


number of vowels in SOCIAL is A, I, O


therefore, number of ways to arrange them so (A, I, O) come together


n(E)= 4! × 3!





Question 23.

The letters of the word ‘CLIFTON’ are placed at random in a row. What is the chance that two vowels come together?


Answer:

given: word “CLIFTON”

formula:


In the random arrangement of the alphabets of word “CLIFTON” we have to find the probability that vowels come together


total possible outcomes of arranging the alphabets are 7!


therefore n(S)=7!


let E be the event that vowels come together


number of vowels in SOCIAL is I, O


therefore, number of ways to arrange them so (I, O) come together


n(E)= 6! × 2!





Question 24.

The letters of the word “FORTUNATES’ are arranged at random in a row. What is the chance that the two ‘T’ come together?


Answer:

given: word “FORTUNATES”

formula:


In the random arrangement of the alphabets of word “FORTUNATES” we have to find the probability that two T’s come together


total possible outcomes of arranging the alphabets are 10!


therefore n(S)=10!


let E be the event that T’s come together


therefore, number of ways to arrange them so (T, T) come together


n(E)= 9! × 2!





Question 25.

A committee of two persons is selected from two men and two women. What is the probability that the committee will have (i) no man? (ii) one man? (iii) two men?


Answer:

given: two men and two women

formula:


committee of two persons is to be formed from two men and two women, therefore


total possible outcomes of selecting two persons is 4C2


therefore n(S)=6


(i) let E be the event that no man is in the committee


E= {W, W}


n(E)= 2C2=1 (only woman)




(ii) let E be the event that one man is present in committee


E= {M W}


n(E)= 2C12C1=4




(iii) let E be the event that two men is in the committee


E= {M, M}


n(E)= 2C2=1 (only men)





Question 26.

If odds in favour of an event be 2: 3, find the probability of occurrence of this event


Answer:

given: odds in favour of event is 2:3

formula:


we have to find the probability of occurrence of this event


total possible outcomes is 2k+3k=5k


therefore n(S)= 5k


let E be the event that it occurs


n(E)=2k


probability of occurrence is





Question 27.

If odds against an events be 7: 9, find the probability of non-occurrence of this event.


Answer:

given: odds against of event is 7:9

formula:


we have to find the probability of non-occurrence of this event


total possible outcomes are 7k+9k=16k


therefore n(S)= 16k


let E be the event that it occurs


n(E)=9k


probability of occurrence is




Therefore, the probability of non-occurrence of the event is





Question 28.

Two balls are drawn at random from a bag containing 2 white, 3 red, 5 green and 4 black balls, one by one without, replacement. Find the probability that both the balls are of different colours.


Answer:

given: 2 white, 3 red, 5 green, 4 black

formula:


two balls are drawn one by one, we have to find the probability that they are of different colours


total possible outcomes are 14C2


therefore n(S)= 14C2=91


let E be the event that all balls are of same colour


E= {WW, RR, GG, BB}


n(E)= 2C2+3C2+5C2+4C2=20


probability of occurrence is




Therefore, the probability of non-occurrence of the event (all balls are different) is





Question 29.

Two unbiased dice are thrown. Find the probability that:

(i) neither a doublet nor a total of 8 will appear

(ii) the sum of the numbers obtained on the two dice is neither a multiple of 2 nor a multiple of 3


Answer:

given: two unbiased dice are thrown

formula:


total possible outcomes of from the dice is 6C16C1


therefore n(S)=36


(i) let E be the event that neither a doublet nor a total of 8 will appear


E’ be the event that a doublet or a total of 8 occurs


E’= {(1,1) (2,2) (3,3) (4,4) (5,5) (6,6) (2,6) (6,2) (3,5) (5,3)}


n(E’) = 10




Therefore P(E) is




(ii) let E be the event that sum of number obtain on the dice is neither a multiple of 2 or 3


E’ be the event that sum of number obtain on the dice is either a multiple of 2 or 3, that is total should be 2,3,4,6,8,9,10,12


E’= {(1,1) (1,2) (2,1) (1,3) (2,2) (3,1) (1,5) (2,4) (3,3) (4,2) (5,1) (2,6) (3,5) (4,4) (5,3) (6,2) (3,6) (4,5) (5,4) (6,3) (4,6) (5,5) (6,4) (6,6)}


n(E’) = 24




Therefore P(E) is





Question 30.

A bag contains 8 red, 3 white and 9 blue balls. If three balls are drawn at random, determine the probability that (i) all the three balls are blue balls 9ii) all the balls are of different colours


Answer:

given: bag which contains 8 red, 3 white, 9 blue balls

formula:


three balls are drawn at random therefore


total possible outcomes of selecting two persons is 20C3


therefore n(S)=20C3=1140


(i) let E be the event that all the balls are blue


E= {B, B, B}


n(E)= 9C3=84




(ii) let E be the event that all balls are of different colour


E= {B W R}


n(E)= 8C13C19C1=216





Question 31.

A bag contains 5 red, 6 white and 7 black balls. Two balls are drawn at random. What is the probability that both balls are red or both are black?


Answer:

given: bag which contains 5 red, 6 white, 7 black balls

formula:


two balls are drawn at random


total possible outcomes are 18C2


therefore n(S)= 18C2=153


let E be the event that both balls are either red or black


n(E)=5C2+7C2=31


probability of occurrence of the event is





Question 32.

If a letter is chosen at random from the English alphabet, find the probability that the letter is (i) a vowel (ii) a constant


Answer:

given: a letter is chosen at random from English alphabet

formula:


total possible outcomes of selecting an alphabet is 26C1


therefore n(S)=26C1=26


(i) let E be the event that a vowel has been chosen


E= {a, e, i, o, u}


n(E)= 5C1=5




(ii) let E be the event that a consonant is chosen


E= all alphabets - {a, e, i, o, u}


n(E)= 21C1=21





Question 33.

In a lottery, a person chooses six different numbers at random from 1 to 20, and if these six numbers match with six numbers already fixed by the lottery committee, he wins the prize. What is the probability of winning the prize in the game?


Answer:

given: six numbers are chosen from 1-20

formula:


we have to find the probability of winning the prize


total possible outcomes of selecting six numbers from 1-20 is 20C6


therefore n(S)= 20C6=38760


let E be the event that all six numbers match with the given number (as winning number is fixed)


n(E)=1


probability of occurrence of the event is





Question 34.

20 cards are numbered from 1 to 20. One card is drawn at random. What is the probability that the number on the cards is:

(i) a multiple of 4?

(ii) not a multiple of 4?

(iii) odd?

(iv) greater than 12?

(v) divisible by 5?

(vi) not a multiple of 6?


Answer:

given: 20 cards numbered from 1-20

formula:


one card is drawn at random therefore total possible outcomes are 20C1


therefore n(S)=20C1=20


(i) let E be the event that the number on the drawn card is a multiple of 4


E= {4, 8, 12, 16, 20}


n(E)= 5C1=5




(ii) let E be the event that the number on the drawn card is not a multiple of 4


E’ be the event that the number on the drawn card is a multiple of 4


E’= {4, 8, 12, 16, 20}


n(E)= 5C1=5




P(E)=1-P(E’)



(iii) let E be the event that the number on the drawn card is odd


E= {1, 3, 5, 7, 9, 11, 13, 15, 17, 19}


n(E)= 10C1=10




(iv) let E be the event that the number on the drawn card is greater than 12


E= {13, 14, 15, 16, 17, 18, 19, 20}


n(E)= 8C1=8




(v) let E be the event that the number on the drawn card is a multiple of 5


E= {5, 10, 15, 20}


n(E)= 4C1=4




(vi) let E be the event that the number on the drawn card is not divisible by 6


let E’ be the event that number on the drawn card is divisible by 6


E’= {6, 12, 18}


n(E’)= 3C1=3




P(E)=1-P(E’)




Question 35.

Two dice are thrown. Find the odds in favour of getting the sum

(i) 4 (b) 5

(iii) What are the odds against getting the sum 6?


Answer:

given: two dices are thrown

formula:


total possible outcomes are 6C16C1


therefore n(S)=62=36


(i) let E be the event that total sum is 4 on dice


E= {(1,3) (3,1) (2,2)}


n(E)= 3C1=3




Therefore, probability of event E’ is


P(E’) =1-P(E)



Odds in favour of getting sum as 4 is P(E):P(E’) =1:11


(ii) let E be the event that total sum is 5 on dice


E= {(1,4) (2,3) (3,2) (4,1)}


n(E)= 4C1=4




Therefore, probability of event E’ is


P(E’) =1-P(E)



Odds in favour of getting sum as 5 is P(E):P(E’) =1:8


(iii) let E be the event that total sum is 6 on dice


E= {(1,5) (2,4) (3,3) (4,2) (5,1)}


n(E)= 5C1=5




Therefore, probability of event E’ is


P(E’) =1-P(E)



Odds against of getting sum as 6 is P(E’):P(E) =31:5



Question 36.

What are the odds in favour of getting a spade if a card is drawn from a well-shuffled deck of cards? What are the odd in favour of getting a king?


Answer:

given: pack of 52 playing cards

formula:


one card is drawn from the given pack of cards, total possible outcomes are 52C1


therefore n(S)=52C1=52


(i) let E be the event of getting a spade


n(E)= 13C1=13




Therefore, probability of event E’ is


P(E’) =1-P(E)



Odds in favour of getting a spade is P(E):P(E’) =1:3


(ii) let E be the event of getting a king


n(E)= 4C1=4




Therefore, probability of event E’ is


P(E’) =1-P(E)



Odds in favour of getting a king is P(E):P(E’) =1:12



Question 37.

A box contains 10 red marbles, 20 blue marbles and 30 green marbles. 5 marbles are drawn at random. From the box, what is the probability that:

(i) al are blue?

(ii) at least one is green?


Answer:

given: box containing 10 red, 20 blue and 30 green marbles

formula:


five marbles are drawn from the given box, total possible outcomes are 60C5


therefore n(S)=60C5


(i) let E be the event of getting all blue balls


n(E)= 20C5




(ii) let E be the event of getting at least one green


let E’ be the event of getting no green


E’= {(5B) (1R 4B) (2R 3B) (3R 2B) (4R 1B) (5R)}


5B=20C5


1R 4B=10C120C4


2R 3B=10C220C3


3R 2B=10C320C2


4R 1B=10C420C1


5R= 10C5


P(E) =1-P(E’)






Question 38.

A box contains 6 red marbles numbered 1 through 6 and 4 white marbles numbered from 12 through 15. Find the probability that a marble drawn is (i) white (ii) white and odd numbered (iii) even numbered (iv) red or even numbered.


Answer:

given: box containing 6 red marbles numbered 1-6, 4 white marbles numbered 12-15

formula:


one marble is drawn from the given box, total possible outcomes are 10C1


therefore n(S)=10C1=10


(i) let E be the event of getting white marble


n(E)= 4C1=4




(ii) let E be the event of getting white marble with odd numbered


E= {13,15}


n(E)= 2




(iii) let E be the event of getting even numbered marble


E= {2, 4, 6, 12, 24}


n(E)= 5




(iv) let E1 be the event of getting red marble


(from (i))


Let E2 be the event of getting even numbered marble


(from (ii))


Therefore (E1ꓵ E2) = red coloured and even numbered


n (E1ꓵ E2) =3



By law of addition P(E1∪ E2) = P(E1)+P(E2)- P(E1ꓵ E2)




Question 39.

A class consists of 10 boys and 8 girls. Thee students are selected at random. What is the probability that the selected group has (i) all boys? (ii) all girls? (iii) 1 boy and 2 girls? (iv) at least one girl? (v) at most one girl?


Answer:

given: class consisting of 10 boys and 8 girls

formula:


three students are selected at random, total possible outcomes are 18C3


therefore n(S)=18C3=816


(i) let E be the event that all are boys


n(E)= 10C3=120




(ii) let E be the event that all are girls


n(E)= 8C3=56




(iii) let E be the event that one boy and two girls are selected


n(E)= 8C110C2=360




(iv) let E be the event that at least one girl is in the group


E= {1,2,3}


n(E)= 8C110C2+8C210C1+8C310C0=696




(v) let E be the event that at most one girl is in the group


E= {0, 1}


n(E)= 8C010C3+8C110C2=480





Question 40.

Five cards are drawn from a well-shuffled pack of 52 cards. Find the probability that all the five cards are hearts.


Answer:

given: pack of 52 playing cards

formula:


five cards are drawn from the given deck


we have to find the probability that all of them are hearts


total possible outcomes of one card from the pack is 52C1


therefore n(S)= 52C5=2598960


let E be the event that all cards belong to hearts


n(E)=13C5=1287


probability of occurrence of the event is





Question 41.

A bag contains tickets numbered from 1 to 20. Two tickets are drawn. Find the probability that (i) both the tickets have prime numbers on them (ii) on one there is a prime number and on the other, there is a multiple of 4.


Answer:

given: bag containing tickets numbered 1-20

formula:


two tickets are drawn at random, total possible outcomes are 20C2


therefore n(S)=20C2=190


(i) let E be the event that both tickets have prime number


E= {2,3,5,7,11,13,17,19}


n(E)= 8C2=28




(ii) let E be the event that one ticket has prime number and other is a multiple of 4


E= {2,3,5,7,11,13,17,19} for prime number


E= {4,8,12,16,20} for multiple of 4


n(E)= 8C15C1=40





Question 42.

An urn contains 7 white, 5 black and 3 red balls. Two balls are drawn at random. Find the probability that 9i) both the balls are red (ii) one ball is red and the other is black (iii) one ball is white.


Answer:

given: urn containing 7 white, 5 black and 3 red

formula:


two balls are drawn at random, total possible outcomes are 15C2


therefore n(S)=18C2=105


(i) let E be the event that both balls are red


n(E)= 3C2=3




(ii) let E be the event that one is red and other is black


n(E)= 3C15C1=15




(iii) let E be the event that one ball is white


n(E)= 8C17C1=56





Question 43.

A and B throw a pair of dice. If A throws 9, find B’s chance of throwing a higher number


Answer:

given: pair of dice

formula:


A and B throw a pair of dice


we have to find the probability that B throw’s a higher number


total possible outcomes are 6C16C1


therefore n(S)= 62=36


let E be the event that A throws 9, and B throws greater than 9


E= {(4,6) (5,5) (5,6) (6,4) (6,5) (6,6)}


n(E)= 6


probability of occurrence of the event is





Question 44.

In a hand at Whist, what is the probability that four kings are held by a specified player?


Answer:

given: game of whist is being played

formula:


we have to find the probability that all four kings are held by a specific player


since in one hand at whist a player has 13 cards


total possible outcomes are 52C13


therefore n(S)= 52C13


let E be the event that a player has 4 kings


n(E)= 48C94C4


probability of occurrence of this event is





Question 45.

Find the probability that in a random arrangement of the letters of the word ‘UNIVERSITY’, the two I’s do not come together.


Answer:

given: word “UNIVERSITY”

formula:


we have to find the probability that two I’s do not come together


total possible outcomes for arrangement of alphabets are 10!


therefore n(S)= 10!


let E be the event that both I’s come together


n(E)= 2! × 9!


probability of occurrence of this event is




Let E’ be the event that both I’s do not come together


Therefore, the probability that two I’s do not come together is


P(E’) =1-P(E)





Exercise 33.4
Question 1.

If A and B be mutually exclusive events associated with a random experiment such that P(A) = 0.4 and P(B) = 0.5, then find :

(i) P(A ∪ B)

(ii)

(iii)

(iv)


Answer:

Given A and B are two mutually exclusive events


And, P(A) = 0.4 P(B) = 0.5


By definition of mutually exclusive events we know that:


P(A ∪ B) = P(A) + P(B)


We have to find-


i) P(A ∪ B) = P(A) + P(B) = 0.5 + 0.4 = 0.9


ii) P(A’ ∩ B’) = P(A ∪ B)’ {using De Morgan’s Law}


⇒ P(A’ ∩ B’) = 1 – P(A ∪ B) = 1 – 0.9 = 0.1


iii) P(A’ ∩ B) = This indicates only the part which is common with B and not A ⇒ This indicates only B.


P(only B) = P(B) – P(A ∩ B)


As A and B are mutually exclusive So they don’t have any common parts ⇒ P(A ∩ B) = 0


∴ P(A’ ∩ B) = P(B) = 0.5


iv) P(A ∩ B’) = This indicates only the part which is common with A and not B ⇒ This indicates only A.


P(only A) = P(A) – P(A ∩ B)


As A and B are mutually exclusive So they don’t have any common parts ⇒ P(A ∩ B) = 0


∴ P(A ∩ B’) = P(A) = 0.4



Question 2.

A and B are two events such that P(A) = 0.54, P(B) = 0.69 and P(A ∩ B) =0.35. Find (i) P(A ∪ B), (ii) (iii) (iv)


Answer:

Given A and B are two events


And, P(A) = 0.54 P(B) = 0.69 P(A ∩ B) = 0.35


By definition of P(A or B) under axiomatic approach we know that:


P(A ∪ B) = P(A) + P(B) – P(A ∩ B)


We have to find-


i) P(A ∪ B) = P(A) + P(B) – P(A ∩ B)


= 0.54 + 0.69 – 0.35 = 0.88


ii) P(A’ ∩ B’) = P(A ∪ B)’ {using De Morgan’s Law}


⇒ P(A’ ∩ B’) = 1 – P(A ∪ B) = 1 – 0.88 = 0.12


iii) P(A ∩ B’) = This indicates only the part which is common with A and not B ⇒ This indicates only A.


P(only A) = P(A) – P(A ∩ B)


∴ P(A ∩ B’) = P(A) - P(A ∩ B) = 0.54 – 0.35 = 0.19


iv) P(A’ ∩ B) = This indicates only the part which is common with B and not A ⇒ This indicates only B.


P(only B) = P(B) – P(A ∩ B)


∴ P(A’ ∩ B) = P(B) – P(A ∩ B) = 0.69 – 0.35 = 0.34



Question 3.

Fill in the blanks in the following table :



Answer:

We need to fill the table:



i) By definition of P(A or B) under axiomatic approach we know that:


P(A ∪ B) = P(A) + P(B) – P(A ∩ B)


Using data from table, we get:


∴ P(A ∪ B) =


ii) By definition of P(A or B) under axiomatic approach we know that:


P(A ∪ B) = P(A) + P(B) – P(A ∩ B)


⇒ P(B) = P(A ∪ B) + P(A ∩ B) – P(A)


Using data from table, we get:


∴ P(B) = 0.6 + 0.25 – 0.35 = 0.5


iii) By definition of P(A or B) under axiomatic approach we know that:


P(A ∪ B) = P(A) + P(B) – P(A ∩ B)


⇒ P(A ∩ B) = P(B) + P(A) - P(A ∪ B)


Using data from table, we get:


∴ P(A ∩ B) = 0.5 + 0.35 – 0.7 = 0.15


Filled table is:




Question 4.

If A and B are two events associated with a random experiment such that P(A) = 0.3, P(B) = 0.4 and P(A ∪ B) = 0.5, find P(A ∩ B).


Answer:

Given A and B are two events


And, P(A) = 0.3 P(B) = 0.5 P(A ∪ B) = 0.5


We need to find P(A ∩ B).


By definition of P(A or B) under axiomatic approach(also called addition theorem) we know that:


P(A ∪ B) = P(A) + P(B) – P(A ∩ B)


∴ P(A ∩ B) = P(A) + P(B) – P(A ∪ B)


⇒ P(A ∩ B) = 0.3 + 0.4 – 0.5 = 0.7 – 0.5 = 0.2


∴ P(A ∩ B) = 0.2



Question 5.

If A and B are two events associated with a random experiment such that P(A) = 0.5, P(B) = 0.3 and P(A ∩ B) = 0.2, find P(A ∪ B).


Answer:

Given A and B are two events


And, P(A) = 0.5 P(B) = 0.3 P(A ∩ B) = 0.2


We need to find P(A ∪ B).


By definition of P(A or B) under axiomatic approach(also called addition theorem) we know that:


P(A ∪ B) = P(A) + P(B) – P(A ∩ B)


∴ P(A ∪ B) = P(A) + P(B) – P(A ∩ B)


⇒ P(A ∪ B) = 0.5 + 0.3 – 0.2 = 0.8 – 0.2 = 0.6


∴ P(A ∪ B) = 0.6



Question 6.

If A and B are two events associated with a random experiment such that P(A ∪ B) = 0.8, P(A ∩ B) = 0.3 and = 0.5 , find P(B).


Answer:

Given A and B are two events


And, P(A’) = 0.5 P(A ∩ B) = 0.3 P(A ∪ B) = 0.8


∵ P(A’) = 1 – P(A) ⇒ P(A) = 1 – 0.5 = 0.5


We need to find P(B).


By definition of P(A or B) under axiomatic approach(also called addition theorem) we know that:


P(A ∪ B) = P(A) + P(B) – P(A ∩ B)


∴ P(B) = P(A ∪ B) + P(A ∩ B) – P(A)


⇒ P(B) = 0.8 + 0.3 – 0.5 = 1.1 – 0.5 = 0.6


∴ P(B) = 0.6



Question 7.

Given two mutually exclusive events A and B such that P(A) = 1/2 and P(B) = 1/3, find P(A or B).


Answer:

Given A and B are two mutually exclusive events


And, P(A) = 1/2 P(B) = 1/3


We need to find P(A ‘or’ B).


P(A or B) = P(A ∪ B)


By definition of mutually exclusive events we know that:


P(A ∪ B) = P(A) + P(B)


∴ P(A ∪ B) = P(A) + P(B)


= 1/2 + 1/3 = 5/6



Question 8.

There are three events A, B, C one of which must and only one can happen, the odds are 8 to 3 against A, 5 to 2 against B, fins the odds against C.


Answer:

As, out of 3 events A,B and C only one can happen at a time which means no event have anything common.


∴ We can say that A , B and C are mutually exclusive events.


By definition of mutually exclusive events we know that:


P(A ∪ B ∪ C) = P(A) + P(B) + P(C)


According to question one event must happen.


This implies A or B or C is a sure event.


∴ P(A ∪ B ∪ C) = 1 …Equation 1


We need to find odd against C


Given,


Odd against A = 8/3




⇒ 8 P(A) = 3 – 3 P(A)


⇒ 11 P(A) = 3


∴ P(A) = …Equation 2


Similarly, we are given with: Odd against B = 5/2




⇒ 5 P(B) = 2 – 2 P(B)


⇒ 7 P(B) = 2


∴ P(B) = …Equation 3


From equation 1,2 and 3 we get:


P(C) = 1 - =


∴ P(C’) = 1 – (34/77) = 43/77


∴ Odd against C =



Question 9.

One of the two events must happen. Given that the chance of one is two-third of the other, find the odds in favour of the other.


Answer:

Let A and B are two events.


As, out of 2 events A and B only one can happen at a time which means no event have anything common.


∴ We can say that A and B are mutually exclusive events.


By definition of mutually exclusive events we know that:


P(A ∪ B) = P(A) + P(B)


According to question one event must happen.


This implies A or B is a sure event.


∴ P(A ∪ B) = P(A) + P(B) = 1 …Equation 1


Given, P(A) = (2/3)P(B)


To find: odds in favour of B




∴ P(B’) = 1 – 3/5 = 2/5


∴ Odd in favour of B =



Question 10.

A card is drawn at random from a well-shuffled deck of 52 cards. Find the probability of its being a spade or a king.


Answer:

As a card is drawn from a deck of 52 cards


Let S denotes the event of card being a spade and K denote the event of card being King.


As we know that a deck of 52 cards contains 4 suits (Heart ,Diamond ,Spade and Club) each having 13 cards. The deck has 4 king cards one from each suit.


We know that probability of an event E is given as-


P(E) =


Where n(E) = numbers of elements in event set E


And n(S) = numbers of elements in sample space.


Hence,


P(S) =


P(K) =


And P(S ∩ K) =


We need to find the probability of card being spade or king, i.e.


P(Spade ‘or’ King) = P(S ∪ K)


Note: By definition of P(A or B) under axiomatic approach(also called addition theorem) we know that:


P(A ∪ B) = P(A) + P(B) – P(A ∩ B)


∴ P(S ∪ K) = P(S) + P(K) - P(S ∩ K)


⇒ P(S ∪ K) =


∴ P(S ∪ K) = 4/13



Question 11.

In a single throw of two dice, find the probability that neither a doublet nor a total of 9 will appear.


Answer:

In a single throw of 2 die, we have total 36(6 × 6) outcomes possible.


Say, n(S) = 36 where S represents Sample space


Let A denotes the event of getting a doublet.


∴ A = {(1,1), (2,2), (3,3), (4,4), (5,5), (6,6)}


∴ P(A) =


And B denotes the event of getting a total of 9


∴ B = {(3,6), (6,3), (4,5), (5,4)}


P(B) =


We need to find probability of the event of getting neither a doublet nor a total of 9.


P(A’ ∩ B’) = ?


As, P(A’ ∩ B’) = P(A ∪ B)’ {using De Morgan’s theorem}


P(A’ ∩ B’) = 1 – P(A ∪ B)


Note: By definition of P(E or F) under axiomatic approach(also called addition theorem) we know that:


P(E ∪ F) = P(E) + P(F) – P(E ∩ F)


∴ P(A ∪ B) = {As P(A ∩ B) = 0 since nothing is common in set A and B ⇒ n(A ∩ B) = 0 }


Hence,


P(A’ ∩ B’) = 1 – (5/18) = 13/18



Question 12.

A natural number is chosen at random from amongst first 500. What is the probability that the number so chosen is divisible by 3 or 5?


Answer:

Given, Sample space is the set of first 500 natural numbers.


∴ n(S) = 500


Let A be the event of choosing the number such that it is divisible by 3


∴ n(A) = [500/3] = [166.67] = 166 {where [.] represents Greatest integer function}


∴ P(A) =


Let B be the event of choosing the number such that it is divisible by 5


∴ n(B) = [500/5] = [100] = 100 {where [.] represents Greatest integer function}


∴ P(B) =


We need to find the P(such that number chosen is divisible by 3 or 5)


∵ P(A or B) = P(A ∪ B)


Note: By definition of P(E or F) under axiomatic approach(also called addition theorem) we know that:


P(E ∪ F) = P(E) + P(F) – P(E ∩ F)


∴ P(A ∪ B) = P(A) + P(B) – P(A ∩ B)


We don’t have value of P(A ∩ B) which represents event of choosing a number such that it is divisible by both 3 and 5 or we can say that it is divisible by 15.


n(A ∩ B) = [500/15] = [33.34] = 33


∴ P(A ∩ B) =


∴ P(A ∪ B) =



Question 13.

A die is thrown twice. What is the probability that at least one of the two throws come up with the number 3?


Answer:

If a dice is thrown twice , it has a total of(6 × 6) 36 possible outcomes.


If S represents the sample space then,


n(S) = 36


Let A represent events the event such that 3 comes in the first throw.


∴ A = {(1,3), (2,3), (3,3), (4,3), (5,3), (6,3)}


⇒ P(A) =


Let B represent events the event such that 3 comes in the second throw.


∴ B = {(3,1), (3,2), (3,3), (3,4), (3,5), (3,6)}


⇒ P(B) =


Clearly (3,3) is common in both events-


∴ P(A ∩ B) =


We need to find the probability of event such that at least one of the 2 throws give 3 i.e. P(A or B) = P(A ∪ B)


Note: By definition of P(E or F) under axiomatic approach(also called addition theorem) we know that:


P(E ∪ F) = P(E) + P(F) – P(E ∩ F)


∴ P(A ∪ B) = P(A) + P(B) – P(A ∩ B)


⇒ P(A ∪ B) =


Hence,


P(at least one of the two throws comes to be 3) =



Question 14.

A card is drawn from a deck of 52 cards. Find the probability of getting an ace or a spade card.


Answer:

As a card is drawn from a deck of 52 cards


Let S denotes the event of card being a spade and K denote the event of card being Ace.


As we know that a deck of 52 cards contains 4 suits (Heart ,Diamond ,Spade and Club) each having 13 cards. The deck has 4 Ace cards one from each suit.


We know that probability of an event E is given as-


P(E) =


Where n(E) = numbers of elements in event set E


And n(S) = numbers of elements in sample space.


Hence,


P(S) =


Similarly,


P(K) =


And P(S ∩ K) =


We need to find the probability of card being spade or king, i.e.


P(Spade ‘or’ Ace) = P(S ∪ K)


Note: By definition of P(A or B) under axiomatic approach(also called addition theorem) we know that:


P(A ∪ B) = P(A) + P(B) – P(A ∩ B)


∴ P(S ∪ K) = P(S) + P(K) - P(S ∩ K)


⇒ P(S ∪ K) =


∴ P(S ∪ K) = 4/13



Question 15.

The probability that a student will pass the final examination in both English and Hindi is 0.5 and the probability of passing neither is 0.1. If the probability of passing the English examination is 0.75. What is the probability of passing the Hindi examination?


Answer:

Let E denotes the event that student passes in English examination.


And H be the event that student passes in Hindi exam.


Given,


P(E) = 0.75


P(passing both) = P(E ∩ H) = 0.5


P(passing neither) = P(E’ ∩ H’) = 0.1


P(H) = ?


As, we know that P(A’ ∩ B’) = P(A ∪ B)’ {using De Morgan’s law}


∴ P(E’ ∩ H’) = P(E ∪ H)’


⇒ 0.1 = 1 – P(E ∪ H)


⇒ P(E ∪ H) = 1 – 0.1 = 0.9


Note: By definition of P(A or B) under axiomatic approach(also called addition theorem) we know that:


P(A ∪ B) = P(A) + P(B) – P(A ∩ B)


∴ P(E ∪ H) = P(E) + P(H) – P(E ∩ H)


⇒ 0.9 = 0.75 + P(H) – 0.5


⇒ 1.4 – 0.75 = P(H)


∴ P(H) = 0.65



Question 16.

One number is chosen from numbers 1 to 100. Find the probability that it is divisible by 4 or 6?


Answer:

Given, Sample space is the set of first 100 natural numbers.


∴ n(S) = 100


Let A be the event of choosing the number such that it is divisible by 4


∴ n(A) = [100/4] = [25] = 25 {where [.] represents Greatest integer function}


∴ P(A) =


Let B be the event of choosing the number such that it is divisible by 6


∴ n(B) = [100/6] = [16.67] = 16 {where [.] represents Greatest integer function}


∴ P(B) =


We need to find the P(such that number chosen is divisible by 4 or 6)


∵ P(A or B) = P(A ∪ B)


Note: By definition of P(E or F) under axiomatic approach(also called addition theorem) we know that:


P(E ∪ F) = P(E) + P(F) – P(E ∩ F)


∴ P(A ∪ B) = P(A) + P(B) – P(A ∩ B)


We don’t have value of P(A ∩ B) which represents event of choosing a number such that it is divisible by both 4 and 6 or we can say that it is divisible by 12.


n(A ∩ B) = [100/12] = [8.33] = 8


∴ P(A ∩ B) =


∴ P(A ∪ B) =



Question 17.

From a well shuffled deck of 52 cards, 4 cards are drawn at random. What is the probability that all the drawn cards are of the same colour.


Answer:

In a deck of 52 cards there are 2 colours. Each colour having 26 cards.


As we need to choose 4 cards out of 52. Let S represents the sample space.


∴ n(S) = 52C4


Let A represents the event that all 4 cards drawn are black.


∴ n(A) = ways in which 4 cards can be selected from 26 black cards.


⇒ n(A) = 26C4


∴ P(A) = 26C4/52C4 =


Let B represents the event that all 4 cards drawn are red.


∴ n(B) = ways in which 4 cards can be selected from 26 red cards.


⇒ n(B) = 26C4


∴ P(B) = 26C4/52C4 =


As we need to find the probability of event such that all drawn cards are from same colour. This means we need to find


P(A ∪ B)


Note: By definition of P(E or F) under axiomatic approach(also called addition theorem) we know that:


P(E ∪ F) = P(E) + P(F) – P(E ∩ F)


∴ P(A ∪ B) = P(A) + P(B) – P(A ∩ B)


As both events A and B have no common elements or we can say that they are mutually exclusive


∴ P(A ∩ B) = 0


Hence,


P(A ∪ B) = P(A) + P(B) =



Question 18.

100 student appeared for two examinations. 60 passed the first, 50 passed the second and 30 passed both. Find the probability that a student selected at random has passed at least one examination.


Answer:

Let E denotes the event that student passed in first examination.


And H be the event that student passed in second exam.


S is the sample space containing the students who appeared for the exam.


Given,


n(S) = 100


n(E) = 60


n(H) = 50


also no of students who passed both exam = n(E ∩ H) = 30


∴ P(E) =


Similarly, P(H) =


And, P(E ∩ H) =


We need to find the probability of event such that a student selected at random has passed at least one examination.


This can be given as – P(E or H) = P(E ∪ H)


Note: By definition of P(A or B) under axiomatic approach(also called addition theorem) we know that:


P(A ∪ B) = P(A) + P(B) – P(A ∩ B)


∴ P(E ∪ H) = P(E) + P(H) – P(E ∩ H)


⇒ P(E ∪ H) =


∴ P(E ∪ H) = 4/5



Question 19.

A box contains 10 white, 6 red and 10 black balls. A ball is drawn at random from the ox. What is the probability that the ball drawn is either white or red?


Answer:

As box contains 26 balls. Let S represents the sample space.


∴ n(S) = 26


Let W denotes the event of drawing a white ball and R represents event of drawing red ball.


As, n(W) = 10 and n(R) = 6


∴ P(W) = 10/26 = 5/13


And P(R) = 6/26 = 3/13


As both events have nothing in common so we can say that they are mutually exclusive.


We need to find the probability of the event such that ball drawn is red or white.


P(Red or White) = P(R ∪ W)


As events are mutually exclusive


∴ P(R ∪ W) = P(R) + P(W)


⇒ P(R ∪ W) = 3/13 + 5/13 = 8/13



Question 20.

In a race, the odds in favour of horses A,B,C,D are 1:3, 1:4, 1:5 and 1:6 respectively. Find probability that one of the wins the race.


Answer:

Given, odds in favour of A is



⇒ 1 – P(A) = 3P(A)


⇒ 4P(A) = 1 ⇒ P(A) = 1/4


Odds in favour of horse B is



⇒ 1 – P(B) = 4P(B)


⇒ 5P(B) = 1 ⇒ P(B) = 1/5


Odds in favour of horse C is



⇒ 1 – P(C) = 5P(C)


⇒ 6P(C) = 1 ⇒ P(C) = 1/6


Odds in favour of horse D is



⇒ 1 – P(D) = 6P(D)


⇒ 7P(D) = 1 ⇒ P(D) = 1/7


We have to find the probability that one of the horses win the race.


∵ only one horse can win the race ⇒ A ,B,C and D are mutually exclusive events.


We need to find P(A ∪ B ∪ C ∪ D).


∵ A ,B,C and D are mutually exclusive events.


∴ P(A ∪ B ∪ C ∪ D) = P(A) + P(B) + P(C) + P(D)


=


Hence,


probability that one of the horses win the race = 319/420



Question 21.

The probability that a person will travel by plane is 3/5 and that he will travel by train is 1/4. What is the probability that he (she) will travel by plane or train?


Answer:

Let T denotes the event that person travels by train and A denotes that event that person travels by plane


Given, P(T) = 3/5 and P(A) = 1/4


We need to find the probability that person travels by plane or train i.e. P(T or A) = P(T ∪ A)


Note: By definition of P(A or B) under axiomatic approach(also called addition theorem) we know that:


P(A ∪ B) = P(A) + P(B) – P(A ∩ B)


∴ P(T ∪ A) = P(T) + P(A) – P(A ∩ T)


∵ A person can never travel with both plane and train simultaneously.


∴ P(A ∩ T) = 0


Hence,


P(T ∪ A) = P(T) + P(A) = 3/5 + 1/4 = 17/20



Question 22.

Two cards are drawn from a well shuffled pack of 52 cards. Find the probability that either both are black or both are kings.


Answer:

As 2 cards are drawn from a deck of 52 cards. This can be done in 52C2 ways. If S represents the sample space,


n(S) = 52C2


Let B represents the event that both drawn cards are black.


∵ A deck of 52 cards has 26 black cards. So 2 cards can be selected out of those 26 in 26C2 ways


∴ n(B) = 26C2


∴ P(B) =


Let K represents the event that both drawn cards are king.


∵ A deck of 52 cards has 4 king cards. So 2 cards can be selected out of those 4 in 4C2 ways


∴ n(K) = 4C2


∴ P(K) =


We need to find the probability that either both are black or both are kings i.e. P(B or K) = P(B ∪ K)


Note: By definition of P(A or B) under axiomatic approach(also called addition theorem) we know that:


P(A ∪ B) = P(A) + P(B) – P(A ∩ B)


∴ P(B ∪ K) = P(B) + P(K) – P(B ∩ K)


As we don’t have value of P(B ∩ K) so we will find it first.


As there is a common element among the events B and K as both the cards can be a king and can be black 2.


∵ 2 black king cards are present so we need to select 2 cards oout of them only. This can be done in 2C2 ways = 1


∴ P(B ∩ K) =


∴ P(B ∪ K) =



Question 23.

In an entrance test that is graded on the basis of two examinations, the probability of a randomly chosen student passing the first examination is 0.8 and the probability of a the second examination is 0.7. The probability of passing at least one of them is 0.95. What is the probability of passing both?


Answer:

Let E denotes the event that student passed in first examination.


And H be the event that student passed in second exam.


Given, P(E) = 0.8 and P(H) = 0.7


Also probability of passing atleast one exam i.e P(E or H) = 0.95


Or, P(E ∪ H) = 0.95


We have to find the probability of the event in which students pass both the examinations i.e. P(E ∩ H)


Note: By definition of P(A or B) under axiomatic approach(also called addition theorem) we know that:


P(A ∪ B) = P(A) + P(B) – P(A ∩ B)


∴ P(E ∪ H) = P(E) + P(H) – P(E ∩ H)


⇒ P(E ∩ H) = P(E) + P(H) – P(E ∪ H)


⇒ P(E ∩ H) = 0.7 + 0.8 – 0.95 = 1.5 – 0.95 = 0.55


∴ Probability of passing both the exams = P(E ∩ H) = 0.55



Question 24.

A box contains 30 bolts and 40 nuts. Half of the bolts and half of the nuts are rusted. If two items are drawn at random, what is the probability that either both are rusted or both are bolts?


Answer:

Let S represents the sample space containing all possible ways of selecting 2 items (out 0f 70 nuts and bolts)


Let R represents the event of drawing 2 rusted item.


And B be the event of drawing 2 bolts


According to question-


n(S) = total ways of selecting 2 items out of 70 = 70C2


∵ half of bolts (30/2 = 15) and half of nuts(40/2 = 20) are rusted


∴ n(R) = no of ways in which 2 rusted items can be drawn.


⇒ n(R) = 35C2


And n(B) = no of ways in which 2 bolts can be drawn.


∴ n(B) = 30C2


Also, n(B ∩ R) = no of ways of selecting 2 items such that they are rusted bolts


∴ n(B ∩ R) = 15C2


Hence, we have –


P(R) =


P(B) =


And P(R ∩ B) =


As we have to find probability of the events such that drawn items are rusted or bolts i.e. P(R ∪ B)


Note: By definition of P(A or B) under axiomatic approach(also called addition theorem) we know that:


P(A ∪ B) = P(A) + P(B) – P(A ∩ B)


∴ P(R ∪ B) = P(R) + P(B) – P(R ∩ B)


⇒ P(R ∪ B) =



Question 25.

An integer is chosen at random from first 200 positive integers. Find the probability that the integer is divisible by 6 or 8.


Answer:

Given, Sample space is the set of first 200 natural numbers.


∴ n(S) = 200


Let A be the event of choosing the number such that it is divisible by 6


∴ n(A) = [200/6] = [33.334] = 33 {where [.] represents Greatest integer function}


∴ P(A) =


Let B be the event of choosing the number such that it is divisible by 8


∴ n(B) = [200/8] = [25] = 25 {where [.] represents Greatest integer function}


∴ P(B) =


We need to find the P(such that number chosen is divisible by 6 or 8)


∵ P(A or B) = P(A ∪ B)


Note: By definition of P(E or F) under axiomatic approach(also called addition theorem) we know that:


P(E ∪ F) = P(E) + P(F) – P(E ∩ F)


∴ P(A ∪ B) = P(A) + P(B) – P(A ∩ B)


We don’t have value of P(A ∩ B) which represents event of choosing a number such that it is divisible by both 4 and 6 or we can say that it is divisible by 24.


n(A ∩ B) = [200/24] = [8.33] = 8


∴ P(A ∩ B) =


∴ P(A ∪ B) =



Question 26.

Find the probability of getting 2 or 3 tails when a coin is tossed four times.


Answer:

When a coin is tossed 4 times. A total of 24 = 16 outcomes are possible.


Let S be the set consisting of all such outcomes.


∴ n(S) = 16


Let A be the event of getting 2 tails.


∴ A = { TTHH,THTH,THHT,HTTH,HTHT,HHTT}


∴ n(A) = 6


∴ P(A) = 6/16 = 3/8


Let B be the event of getting 3 tails.


∴ B = { TTTH ,TTHT, THTT,HTTT }


⇒ n(B) = 4


∴ P(B) = 4/16 = 1/4


We need to find the probability of getting 2 tails or 3 tails i.e.


P(A ∪ B) = ?


As we can’t get 2 and 3 tails at the same time. So A and B are mutually exclusive events.


∴ P(A ∪ B) = P(A) + P(B) = 3/8 + 1/4 = 5/8



Question 27.

Suppose an integer from 1 through 1000 is chosen at random, fins the probability that the integer is a multiple of 2 or a multiple of 9.


Answer:

Given, Sample space is the set of first 1000 natural numbers.


∴ n(S) = 1000


Let A be the event of choosing the number such that it is multiple of 2


∴ n(A) = [1000/2] = [500] = 500 {where [.] represents Greatest integer function}


∴ P(A) =


Let B be the event of choosing the number such that it is multiple of 9


∴ n(B) = [1000/9] = [111.11] = 111 {where [.] represents Greatest integer function}


∴ P(B) =


We need to find the P(such that number chosen is multiple of 2 or 9)


∵ P(A or B) = P(A ∪ B)


Note: By definition of P(E or F) under axiomatic approach(also called addition theorem) we know that:


P(E ∪ F) = P(E) + P(F) – P(E ∩ F)


∴ P(A ∪ B) = P(A) + P(B) – P(A ∩ B)


We don’t have value of P(A ∩ B) which represents event of choosing a number such that number is a multiple of both 2 and 9 or we can say that it is a multiple of 18.


n(A ∩ B) = [1000/18] = [55.55] = 55


∴ P(A ∩ B) =


∴ P(A ∪ B) =



Question 28.

In a large metropolitan area, the probabilities are 0.87, 0.36, 0.30 that a family (randomly chosen for a sample survey) owns a colour television set, a black and white television set, or both kinds of sets. What is the probability that a family owns either any one or both kinds of sets?


Answer:

Let C represents the event that random family has colour television, W represents the event that family has black & white set and C ∩ W represents that they own both kind of sets.


According to question:


P(C) = 0.87


P(W) = 0.36 and P(C ∩ W) = 0.30


We need to find the probability of the event that a family owns either anyone kind of set.


i.e we need to find P(C or W) = P(C ∪ W) = ?


Note: By definition of P(E or F) under axiomatic approach(also called addition theorem) we know that:


P(E ∪ F) = P(E) + P(F) – P(E ∩ F)


∴ P(C ∪ W) = P(C) + P(W) – P(C ∩ W)


⇒ P(C ∪ W) = 0.87 + 0.36 – 0.30 = 0.93



Question 29.

If A and B are mutually exclusive events such that P(A) = 0.35 and P(B) = 0.45, find

(i) P (A ∪ B) (ii) P (A ∩ B)

(iii) (iv)


Answer:

Given A and B are two mutually exclusive events


And, P(A) = 0.35 P(B) = 0.45


By definition of mutually exclusive events we know that:


P(A ∪ B) = P(A) + P(B)


We have to find-


i) P(A ∪ B) = P(A) + P(B) = 0.35 + 0.45 = 0.8


ii) P(A ∩ B) = 0 {∵ nothing is common between A and B}


iii) P(A ∩ B’) = This indicates only the part which is common with A and not B ⇒ This indicates only A.


P(only A) = P(A) – P(A ∩ B)


As A and B are mutually exclusive So they don’t have any common parts ⇒ P(A ∩ B) = 0


∴ P(A ∩ B’) = P(A) = 0.35


iv) P(A’ ∩ B’) = P(A ∪ B)’ {using De Morgan’s Law}


⇒ P(A’ ∩ B’) = 1 – P(A ∪ B) = 1 – 0.8 = 0.2



Question 30.

A sample space consists of 9 elementary event E1, E2, E3, ……E8, E9 whose probabilities are P(E1) = P(E2) = 0.08, P(E3) = P(E4) = 0.1, P(E6) = P(E7) = 0.2, P(E8) = P(E9) = 0.07

Suppose A = {E1, E5, E8}, B = {E2, E8, E9}

(i) Compute P(A), P(B) and P(A ∩ B).

(ii) Using the addition law of probability, find P (A ∪ B).

(iii) List the composition of the event A ∪ B, and calcite P(A ∪ B) by adding the probabilities of the elementary events.

(iv) Calculate from P(B), also calculate directly from the elementary events of .


Answer:

Clearly according to questions sample space contains 9 elementary events(events with single outcome)


Let S represents the sample space.


∴ S = E1∪ E2∪ E3 ∪ ……∪ E8∪ E9


Given A = {E1, E5, E8}


Or A = E1∪ E5∪ E8


P(E1) = P(E2) = 0.08, P(E3) = P(E4) = 0.1, P(E6) = P(E7) = 0.2, P(E8) = P(E9) = 0.07


∴ P(A) = P(E1∪ E5∪ E8) = P(E1)+P(E5) + P(E8)


⇒ P(A) = 0.08 + P(E5) + 0.07 = 0.15 + P(E5) …(1)


P(E5) is missing, so we need to find it.


Given,


B = {E2, E8, E9} or B = E2∪ E8∪ E9


∴ P(B) = P(E2∪ E8∪ E9) = P(E2)+P(E8) + P(E9) = = 0.08+0.07+0.07 = 0.21


∴ P(B) = 0.21 ….ans(i)


∴ B’ = {E1, E3, E4, E5, E6, E7} or B’ = E1∪ E3∪ E4∪ E5∪ E6∪ E7


∴ P(B’) = P(E1) + P(E3) + P(E4) + P(E5) + P(E6) + P(E7)


1 – 0.21 = 0.08 + 0.1 + 0.1 + P(E5) + 0.2 + 0.2


⇒ 0.79 = 0.68 + P(E5)


∴ P(E5) = 0.79 – 0.68 = 0.11


∴ from equation 1, we get-


P(A) = 0.15 + P(E5) = 0.15 + 0.11 = 0.26 (i)


Clearly A ∩ B = {E8}


∴ P(A ∩ B) = P(E8) = 0.07 (i)


Using addition law of probability we know that-


P(A ∪ B) = P(A) + P(B) – P(A ∩ B)


⇒ P(A ∪ B) = 0.26 + 0.21 – 0.07


∴ P(A ∪ B) = 0.4 (ii)


As, A ∪ B = {E1, E5, E8} ∪ {E2, E8, E9}


⇒ A ∪ B = {E1, E5, E8, E2, E9}


∴ P(A ∪ B) = P(E1)+ P(E5)+ P(E8)+ P(E2)+ P(E9)


⇒ P(A ∪ B) = 0.08 + 0.11 + 0.07 + 0.08 + 0.07 = 0.41


∴ P(A ∪ B) = 0.41 (iii)


As , P(B) = 0.21


∴ P(B’) = 1 – 0.21 = 0.79 (iv)


Calculation of P(B’) using sets –


B’ = {E1, E3, E4, E5, E6, E7} or B’ = E1∪ E3∪ E4∪ E5∪ E6∪ E7


∴ P(B’) = P(E1) + P(E3) + P(E4) + P(E5) + P(E6) + P(E7)


P(B’) = 0.08 + 0.1 + 0.1 + 0.11 + 0.2 + 0.2


= 0.79 (iv)


Clearly through both the ways we get the same answer.




Very Short Answer
Question 1.

Three numbers are chosen at random from numbers 1 to 30. Write the probability that the chosen numbers are consecutive.


Answer:

Let E denote the event that the chosen numbers are consecutive.


No of ways in which 3 numbers can be chosen out of 30 = 30C3


As we have to select 3 consecutive numbers, if we select 1 number other two are already selected.


As 29,30 can’t be selected because if they are selected we won’t be able to get 3 consecutive numbers.


∴ number of ways in which 3 consecutive numbers can be selected = number of ways in which 1 number can be chosen out of numbers from 1 to 28 = 28C1 ways


∴ P(E) =


Thus, P(E) =



Question 2.

n (> 3) persons are sitting in a row. Two of them are selected. Write the probability that they are together.


Answer:

Let E denote the event that the selected persons are sitting together.


As 2 persons can be selected out of n in nC2 ways


Out of n persons we can select two persons sitting together in (n-1) ways.


Because we have to select only one person next person is going to be automatically selected.


We can’t select last person because no one is sitting next to him.


∴ 1 person out of n-1 persons can be selected in (n-1) ways.


∴ P(E) =


Thus, P(E) =



Question 3.

A single letter is selected at random from the word ‘PROBABILITY’. What is the probability that it is a vowel?


Answer:

As there are 9 distinct letters in “PROBABILITY” (P,R,O,B,A,I,L,T,Y).


∴ A single letter can be selected in 9 ways.


As there are 3 vowels in “PROBABILITY”.


∴ 1 vowel can be selected in 3 ways


∴ P(selected letter is vowel) = 3/9 = 1/3



Question 4.

What is the probability that a leap year will have 53 Fridays or 53 Saturday?


Answer:

In a leap year we have 366 days. So everyday of a week comes 52 times in 364 days.


Now we have 2 days remaining.


These 2 days can be-


S = {MT, TW, WTh, ThF, FSa, SaSu, SuM}


Where S is the sample space.


As there are total 7 possibilities.


∴ n(S) = 7


Let A denote the event of getting 53 Fridays and B denote event of getting 53 Saturdays.


We have to find P(A ∪ B)


Clearly,


P(A) = 2/7


P(B) = 2/7


And P(A ∩ B) = 1/7


∴ P(A ∪ B) = 2/7 + 2/7 – 1/7 = 3/7



Question 5.

Three dice are thrown simultaneously. What is the probability of getting 15 as the sum?


Answer:

As 3 dice are thrown. So total possible outcomes = 6×6×6 = 216


For getting 15 as the sum we need this combination-


(5, 5, 5) or (6,4,5) or (6,6,3)


(6, 4, 5) can be arranged in 3! = 6 ways.


(6, 4, 5) can be arranged in (3!)/(2!) = 3 ways


∴ 10 favourable outcomes are possible.


∴ P(E) = 10/216



Question 6.

If the letters of the word ‘MISSISSIPPI’ are written down at random in a row, what is the probability that four S’s come together.


Answer:

Letters of MISSISSIPPI has 11 letters. Out of which 4 S’s are repeating, 4 I’s are repeating and 2 P’s are also repeating.


By Permutation theory.


Letters of MISSISSIPPI can be arranged in


Take 4 S’s together and treat them as single letter. Now we have


8 letters in together – MI(SSSS)IIPPI → MI(X)IIPPI


Letters of MI(X)IIPPI can be arranged in


∴ P(4’S come together) =



Question 7.

What is the probability that the 13th days of a randomly chosen month is Friday?


Answer:

13th day is going to be any of the day from Monday to Sunday. So probability that 13th day of a month being Friday is 1/7.

But here the month is not declared. So the probability of selecting a random month is 1/12


By multiplication theory of Probability -


∴ probability that the 13th days of a randomly chosen month is Friday = (1/7)×(1/12) = 1/84



Question 8.

Three of the six vertices of a regular hexagon are chosen at random. What is the probability that the triangle with these vertices is equilateral.


Answer:

3 vertices out of 6 can be chosen in 6C3 = 20 ways


In a hexagon only 2 equilateral triangles are possible.


∴ probability that the triangle with these vertices is equilateral = 2/20 = 1/10



Question 9.

If E and E2 are independent events, write the value of


Answer:

As E1 and E2 are independent events.


∴ P((E1∪ E2)∩(E1’ ∩ E2’))


As by De Morgan’s law.


E1’ ∩ E2’ = (E1∪ E2)’


∴ P((E1∪ E2)∩(E1’ ∩ E2’)) = P((E1∪ E2)∩(E1∪ E2)’)


⇒ P((E1∪ E2)∩(E1∪ E2)’) = P(empty set) = 0



Question 10.

If A and B are two independent events such that , then write the values of P(A) and P(B).


Answer:

As A and B are independent events.


∴ P(A ∩ B) = P(A)P(B) = 1/6


Also, P(A’ ∩ B’) = P(A’)P(B’) = 1/3


⇒ (1 – P(A))(1 – P(B)) = 1/3


⇒ 1 + P(A)P(B) – P(A) – P(B) = 1/3


⇒ 1 + 1/6 – 1/3 = P(A) + P(B)


⇒ 1 – 1/6 = P(A) + P(B)


∴ P(A) = 5/6 – P(B)


Let P(B) = x


∴ (5/6 – x)x = 1/6


⇒ 5x – 6x2 = 1


⇒ 6x2 – 5x + 1 = 0


⇒ 6x2 – 3x – 2x + 1 = 0


⇒ 3x(2x – 1) – (2x – 1) = 0


⇒ (3x – 1)(2x – 1)=0


∴ x = 1/3 or x = 1/2


∴ P(B) = 1/3 or 1/2


∴ P(A) = 1/2 or 1/3




Mcq
Question 1.

One card is drawn from a pack of 52 cards. The probability that it is the card of a king or spade is
A.

B.

C.

D.


Answer:

As a card is drawn from a deck of 52 cards


Let S denotes the event of card being a spade and K denote the event of card being King.


As we know that a deck of 52 cards contains 4 suits (Heart ,Diamond ,Spade and Club) each having 13 cards. The deck has 4 king cards one from each suit.


We know that probability of an event E is given as-


P(E) =


Where n(E) = numbers of elements in event set E


And n(S) = numbers of elements in sample space.


Hence,


P(S) =


P(K) =


And P(S ∩ K) =


We need to find the probability of card being spade or king, i.e.


P(Spade ‘or’ King) = P(S ∪ K)


Note: By definition of P(A or B) under axiomatic approach(also called addition theorem) we know that:


P(A ∪ B) = P(A) + P(B) – P(A ∩ B)


∴ P(S ∪ K) = P(S) + P(K) - P(S ∩ K)


⇒ P(S ∪ K) =


∴ P(S ∪ K) = 4/13


∴ Option (c) is the correct choice.


Question 2.

Two dice are thrown together. The probability that at least one will show its digit greater than 3 is
A.

B.

C.

D.


Answer:

If a dice is thrown twice, it has a total of (6 × 6) 36 possible outcomes.


If S represents the sample space then,


n(S) = 36


Let A represent events the event such that digit greater than 3 comes in the second throw.


∴ A = {(1,4), (2,4), (3,4), (4,4), (5,4), (6,4), (1,5), (2,5), (3,5), (4,5), (5,5), (6,5), (1,6), (2,6), (3,6), (4,6), (5,6), (6,6)}


⇒ P(A) =


Let B represent events the event such digit greater than 3 comes in the first throw.


Similarly 18 outcomes are possible as were present for event A


⇒ P(B) =


Clearly (4,4), (5,4), (6,4), (4,5), (5,5), (6,5) (4,6), (5,6) and (6,6) are common in both events-


∴ P(A ∩ B) =


We need to find the probability of event such that at least one of the 2 throws give digit greater than 33 i.e. P(A or B) = P(A ∪ B)


Note: By definition of P(E or F) under axiomatic approach(also called addition theorem) we know that:


P(E ∪ F) = P(E) + P(F) – P(E ∩ F)


∴ P(A ∪ B) = P(A) + P(B) – P(A ∩ B)


⇒ P(A ∪ B) =


Hence,


P(at least one of the two throws shows digit >3) =


∴ Option (b) is correct choice.


Question 3.

Two dice are thrown simultaneously. The probability of obtaining a total score of 5 is
A.

B.

C.

D. none of these


Answer:

As 2 dice are thrown so there are 6×6 = 36 possibilities.


Let E denote the event of getting a total score of 5.


E = {(1,4),(2,3),(3,2),(4,1)}


∴ n(E)=4


Hence,


P(E) = 4/36 = 1/9


As our answer matches only with option (c)


∴ Option (c) is the only correct choice.


Question 4.

Two dice are thrown simultaneously. The probability of obtaining total score of seven is
A.

B.

C.

D.


Answer:

As 2 dice are thrown so there are 6×6 = 36 possibilities.


Let E denote the event of getting a total score of 5.


E = {(1,6),(2,5),(3,4),(4,3),(5,2),(6,1)}


∴ n(E)= 6


Hence,


P(E) = 6/36


As our answer matches only with option (b)


∴ Option (b) is the only correct choice.


Question 5.

The probability of getting a total of 10 in a single throw of two dice is
A.

B.

C.

D.


Answer:

As 2 dice are thrown so there are 6×6 = 36 possibilities.


Let E denote the event of getting a total score of 5.


E = {(4,6),(5,5),(6,4)}


∴ n(E)= 3


Hence,


P(E) = 3/36 = 1/12


As our answer matches only with option (b)


∴ Option (b) is the only correct choice.


Question 6.

A card is drawn at random from a pack of 100 cards numbered 1 to 100. The probability of drawing a number which is a square is
A.

B.

C.

D. none of these


Answer:

As a card is to be drawn from 1 to 100.


∴ n(S) = 100


As we know that largest no less that or equal to 100 which is a perfect square = 100


And, √100 = 10


∴ there are total 10 numbers from 1 to 100 which are square.


∴ n(E) = 10


Hence,


P(E) = 10/100 = 1/10


As our answer matches only with option (c)


∴ Option (c) is the only correct choice.


Question 7.

A bag contains 3 red, 4 white and 5 blue balls. All balls are different. Two balls are drawn at random. The probability that they are of different colour is
A.

B.

C.

D. 1


Answer:

As there are 3 +4+5 = 12 balls.


So, 2 balls out of 12 can be drawn in 12C2 ways = 66


Let E denote the event that balls drawn are of different colours


Either the balls will be red-white or white-blue or red-blue.


∴ P(E) =


Hence,


P(E) = 47/66


As our answer matches only with option (a)


∴ Option (a) is the only correct choice.


Question 8.

Two dice are thrown together. The probability that neither they show equal digits nor the sum of their digits is 9 will be
A.

B.

C.

D.


Answer:

In a single throw of 2 die, we have total 36(6 × 6) outcomes possible.


Say, n(S) = 36 where S represents Sample space


Let A denotes the event of getting a doublet(equal number)


∴ A = {(1,1), (2,2), (3,3), (4,4), (5,5), (6,6)}


∴ P(A) =


And B denotes the event of getting a total of 9


∴ B = {(3,6), (6,3), (4,5), (5,4)}


P(B) =


We need to find probability of the event of getting neither a doublet nor a total of 9.


P(A’ ∩ B’) = ?


As, P(A’ ∩ B’) = P(A ∪ B)’ {using De Morgan’s theorem}


P(A’ ∩ B’) = 1 – P(A ∪ B)


Note: By definition of P(E or F) under axiomatic approach(also called addition theorem) we know that:


P(E ∪ F) = P(E) + P(F) – P(E ∩ F)


∴ P(A ∪ B) = {As P(A ∩ B) = 0 since nothing is common in set A and B ⇒ n(A ∩ B) = 0 }


Hence,


P(A’ ∩ B’) = 1 – (5/18) = 13/18


Hence,


P(required event) = 13/18


As our answer matches only with option (b)


∴ Option (b) is the only correct choice.


Question 9.

Four persons are selected at random out of 3 men, 2 women and 4 children. The probability that there are exactly 2 children in the selection is
A.

B.

C.

D. none of these


Answer:

As there are 3 +2+4 = 9 persons


So, 4 persons out of 9 can be drawn in 9C4 ways = 126


Let E denote the event that there are exactly 2 children in the selection.


2children out of 4 can be selected in 4C2 = 6 ways


And rest two persons can be male or female. So we will select 2 persons out of remaining 5.


∴ P(E) =


Hence,


P(E) = 10/21


As our answer matches only with option (c)


∴ Option (c) is the only correct choice.


Question 10.

The probabilities of happening of two events. A and B are 0.25 and 0.50 respectively. If the probability of happing of A and B together is 0.14, then probability that neither A nor B happens is
A. 09.39

B. 0.25

C. 0.11

D. none of these


Answer:

Given, P(A) = 0.25 and P(B) = 0.5


Also P(A ∩ B) = 0.14


We have to find P(A’ ∩ B’)


By De Morgan’s theorem we know that:


P(A’ ∩ B’) = P(A ∪ B)’


We know that P(A ∪ B) = P(A) + P(B) – P(A ∩ B)


∴ P(A ∪ B) = 0.25 + 0.5 – 0.14 = 0.61


∴ P(A’ ∩ B’) = P(A ∪ B)’ = 1 -P(A ∪ B) = 1 – 0.61 = 0.39


As our answer does not match with any other option


But option of none of these is there. So most suitable option is option (d)


∴ Option (d) is the only correct choice.


Question 11.

A die is rolled, then the probability that a number 1 or 6 may appear is
A.

B.

C.

D.


Answer:

When a die is rolled there are in total 6 possibilities.


∴ n(S) = 6


Let E denote event of getting 1 or 6


∴ n(E) = 2


∴ P(E) = 2/6 = 1/3


Hence,


P(E) = 1/3


As our answer matches only with option (c)


∴ Option (c) is the only correct choice.


Question 12.

Six boys and six girls sit in a row randomly. The probability that all girls sit together is
A.

B.

C.

D.


Answer:

As 6 boys and 6 girls are sitting in a row. So these 12 persons can sit in 12! Ways


Now group all 6 girls together and treat them as 1.


Now, all girls together can sit in 7! Ways


And girls can sit among self in 6! Ways.


∴ total ways in which all 6 girls sit together = 7! × 6!


∴ P(E) =


Hence,


P(E) = 1/132


As our answer matches only with option (d)


∴ Option (d) is the only correct choice.


Question 13.

The probabilities of three mutually exclusive events A, B and C are given by 2/3, 1/4 and 1/6 respectively. The statement
A. is true

B. is false

C. nothing can be said

D. could be either


Answer:

As we know that 3 events are said to be mutually exclusive events if –


P(A ∪ B ∪ C) = P(A) + P(B) + P(C)


⇒ P(A ∪ B ∪ C) =


As probability can never be > 1


∴ A,B and C are not mutually exclusive events.


Statement is false


∴ Option (b) is the only correct choice.


Question 14.

If are the probabilities of three mutually exclusive and exhaustive events, then the set of all values of p is
A. (0, 1)

B. (-1/4, 1/3)

C. (0, 1/3)

D. (0, ∞)


Answer:

As 3 events are mutually exclusive and exhaustive.


∴ P(A ∪ B ∪ C) = P(A) + P(B) + P(C) = 1




⇒ 5 + 3p = 6


∴ p = 1/3


As 1/3 lies in both intervals given in option (a) and (d)


But a larger value like 100 which comes if we choose option d can make P(A) < 0 which is not possible.


∴ Option (a) is the most suitable choice here.


Question 15.

A pack of cards contains 4 aces, 4 kings, 4 queens and 4 jacks. Two cards are drawn at random. The probability that at least one of them is an ace is
A.

B.

C.

D.


Answer:

As total number of cards = 16


2 cards can be drawn in 16C2 ways = 120 ways


As we need to find the probability for the event E such that at least one of them is an ace.


We can solve this problem using negation.


We will find the probability for event that both cards are ace (E’)


Automatically 1 – P(E’) will give P(E)


For finding P(E’) we will select both cards out of 4 ace cards


∴ P(E’) =


∴ P(E) = 1 – 1/20 = 19/20


Hence,


P(E) = 19/20


As our answer matches only with option (c)


∴ Option (c) is the only correct choice.


Question 16.

If three dice are thrown simultaneously, then the probability of getting a score of 5 is
A. 5/216

B. 1/6

C. 1/36

D. none of these


Answer:

As 3 dice are thrown so there are 6×6×6 = 216 possibilities.


Let E denote the event of getting a total score of 5.


{(1,1,3),(1,2,2)}


As (1,1,3) can be arranged in


and (1,2,2) can be arranged in


∴ n(E) = 6


Hence,


P(E) = 6/36 = 1/6


As our answer matches only with option (b)


∴ Option (b) is the only correct choice.


Question 17.

One of the two events must occur. If the chance of one is 2/3 of the other, then odds in favour of the other are
A. 1 : 3

B. 3 : 1

C. 2 : 3

D. 3 : 2


Answer:

Let E and F be the two events such that one must occur.


Given,


P(E) = 2/3 P(F)


Also, P(E ∪ F) = 1


P(E) + P(F) = 1


⇒ P(F){2/3 + 1} = 1


∴ P(F) = 3/5


And P(F’) = 1 – 3/5 = 2/5


We have to find


∴ Odds in favour of F = 3/2


As our answer matches only with option (d)


∴ Option (d) is the only correct choice.


Question 18.

The probability that a leap year will have 53 Fridays or 53 Saturdays is
A. 2/7

B. 3/7

C. 4/7

D. 1/7


Answer:

In a leap year we have 366 days. So, every day of a week comes 52 times in 364 days.


Now we have 2 days remaining.


These 2 days can be-


S = {MT, TW, WTh, ThF, FSa, SaSu, SuM}


Where S is the sample space.


As there are total 7 possibilities.


∴ n(S) = 7


Let A denote the event of getting 53 Fridays and B denote event of getting 53 Saturdays.


We have to find P(A ∪ B)


Clearly,


P(A) = 2/7


P(B) = 2/7


And P(A ∩ B) = 1/7


∴ P(A ∪ B) = 2/7 + 2/7 – 1/7 = 3/7


As our answer matches only with option (b)


∴ Option (b) is the only correct choice.


Question 19.

A person write 4 letters and addresses 4 envelopes. If the letters are placed in the envelopes at random, then the probability that all letters are not placed in the right envelopes, is
A. 1/4

B. 11/24

C. 15/24

D. 3/8


Answer:

As we have 4 letters and 4 envelopes.


These 4 letters can be arranged in 4! = 24 ways.


∴ n(S) = 24


Let E denotes the event that all letters are not placed in the right envelopes


The number of ways in which 4 letters can be placed in wrong envelopes is given by the number of ways in which N objects can be dearranged.


Numbers of ways of in which N objects can be dearranged is given by –



∴ n(E) =


∴ P(E) = 9/24 = 3/8


Question 20.

A and B are two events such that P(A) = 0.25 and P(B) = 0.50. The probability of both happening together is 0.14. The probability of both A and B not happening is
A. 0.39

B. 0.25

C. 0.11

D. none of these


Answer:

Given, P(A) = 0.25 and P(B) = 0.5


Also P(A ∩ B) = 0.14


We have to find P(A’ ∩ B’)


By De Morgan’s theorem we know that:


P(A’ ∩ B’) = P(A ∪ B)’


We know that P(A ∪ B) = P(A) + P(B) – P(A ∩ B)


∴ P(A ∪ B) = 0.25 + 0.5 – 0.14 = 0.61


∴ P(A’ ∩ B’) = P(A ∪ B)’ = 1 -P(A ∪ B) = 1 – 0.61 = 0.39


As our answer matches with option (a)


∴ Option (a) is the only correct choice.


Question 21.

If the probability of A to fail in an examination is and that of B is . Then, the probability that either A or B fails is
A. 1/2

B. 11/25

C. 19/50

D. none of these


Answer:

Let A be the event that student ‘A’ fails in exam and B be the event that student ‘B’ fails in exam.


Given, P(A) = 1/5 and P(B) = 3/10


We have to find P(A ∪ B)


We know that P(A ∪ B) = P(A) + P(B) – P(A ∩ B)


As P(A ∩ B) not given, so assuming its value 0


∴ P(A ∪ B) = 1/5 + 3/10 = 5/10 = 1/2


As our answer matches with option (a)


∴ Option (a) is the only correct choice.


Question 22.

A box contains 10 good articles and 6 defective articles. One item is drawn at random. The probability that it is either good or has a defect, is
A. 64/64

B. 49/64

C. 40/64

D. 24/64


Answer:

As total articles = 10 + 6 = 16


P(good) =P(G) = 10/16


And P(defective)= P(D) = 6/16


As there it is not possible to select an item that is both good and defective.


P(good or Defective) = P(G ∪ D) = P(G) + P(D) = 16/16 = 1


As our answer does not directly seems to match with any option.


But option (a) is 64/64 = 1


∴ option (a) is the only correct choice.


Question 23.

Three integers are chosen at random from the first 20 integers. The probability that their product is even is
A. 2/19

B. 2/29

C. 17/19

D. 4/19


Answer:

3 integers out of 20 can be chosen in 20C3 ways.


As in first 20 integers, 10 are even.


To have the product of 3 integer even the chosen integers must be even.


∴ 3 integers that are even can be chosen from first 20 integers in 10C3 ways.


∴ Probability =


Our answer matches with option (a)


∴ Option (a) is the only correct choice


Question 24.

Out of 30 consecutive integers, 2 are chosen at random. The probability that their sum is odd, is
A. 14/29

B. 16/29

C. 15/29

D. 10/29


Answer:

2 integers out of 30 can be chosen in 30C2 ways.


As in first 30 integers, 15 are even and 15 are odd.


To have the sum of 2 integer odd one integer must be even and other must be odd.


∴ integers can be chosen in 15C1 × 15C1 ways


∴ Probability =


Our answer matches with option (c)


∴ Option (c) is the only correct choice


Question 25.

A bag contains 5 black balls, 4 white balls and 3 red balls. If a ball is selected randomly. The probability that it is black or red ball is
A. 1/3

B. 1/4

C. 5/12

D. 2/3


Answer:

According to question bag contains 5+4+3 = 12 balls


∴ 1 ball can be chosen in 12C1 = 12 ways


Let B denotes the event of drawing a black ball and R denotes event of drawing red ball


∴ P(B) = 5/12 and P(R) = 3/12


We have to find P(B ∩ R)


As, we need to find P(B ∪ R)


∵ Black and red balls can’t be drawn simultaneously.


∴ P(B ∩ R) = 0


∴ P(B ∪ R) = P(B) + P(R) = 5/12 + 3/12 = 8/12 = 2/3


Our answer matches with option (d)


∴ Option (d) is the only correct choice


Question 26.

Two dice are thrown simultaneously. The probability of getting a pair of sixes is
A. 1/36

B. 1/3

C. 1/6

D. none of these


Answer:

Sample Space = 36


Out of 36 possible events, only one event is favourable = (6, 6)


Probability


Question 27.

An urn contains 9 balls two of which are red, three blue and four black. Three balls are drawn at random. The probability that they are of the same colour is
A. 5/84

B. 3/9

C. 3/7

D. 7/17


Answer:

Balls in urn = 9


3 balls can be drawn in 9C3 ways.


To draw the balls of same colour, we can select balls all three red, all blue or all black.


P(ball of same colour) = P(all blue) or P(all black) or P(all red)


P(all red) = not possible as we have to select 3 balls but there are only 2 red balls.


∴ P(All red) = 0


P(all black) =


And P(all blue) =


∴ P(ball of same colour) = P(all blue) or P(all black) or P(all red)


⇒ P(ball of same colour) = P(all blue) + P(all black) + P(all red)


= 1/84 + 1/21 + 0 = 5/84


Our answer matches with option (a)


∴ Option (a) is the only correct choice


Question 28.

Five persons entered the lift cabin on the ground floor of an 8 floored house. Suppose that each of them independently and with equal probability can leave the cabin at any floor beginning with the first, then the probability of all 5 persons leaving at different floor is
A.

B.

C.

D.


Answer:

As building has 8 floors including ground.


So, they have 7 options to leave the lift.


∴ total ways in which persons can leave the lift = 75


As the number of ways in which all persons leave in a different floor can be given by: 7× 6× 5× 4× 3 = 7P5


∴ P(that all leave lift in different floor) =


Our answer matches with option (a)


∴ Option (a) is the only correct choice


Question 29.

A box contains 10 good articles and 6 with defects. One item is drawn at random. The probability that it is either good or has a defect is
A. 64/64

B. 49/64

C. 40/64

D. 24/64


Answer:

As total articles = 10 + 6 = 16


P(good) =P(G) = 10/16


And P(defective)= P(D) = 6/16


As there it is not possible to select an item that is both good and defective.


P(good or Defective) = P(G ∪ D) = P(G) + P(D) = 16/16 = 1


As our answer does not directly seems to match with any option.


But option (a) is 64/64 = 1


∴ option (a) is the only correct choice.


Question 30.

A box contains 6 nails and 10 nuts. Half of the nails and half of the nuts are rusted. If one item is chosen at random, the probability that it is rusted or is a nail is
A. 3/16

B. 5/16

C. 11/16

D. 14/16


Answer:

As total articles(nuts + nails) = 10 + 6 = 16


Number of rusted nails = 3


Number of rusted nuts = 5


Number of rusted articles = 8


Let N denotes the event that the article drawn is a nails and R be the event denoting drawn article is rusted


P(N) = 6/16


And P(R) = 8/16


Also, we have possibility that drawn nail is rusted.


∴ P(N ∩ R) = 3/16


P(Nail or Rusted) = P(N ∪ R) = P(N) + P(R) – P(N ∩ R) = 6/16 + 8/16 – 3/16 = 11/16


∴ option (c) is the only correct choice.


Question 31.

If S is the sample space and , where A and B are two mutually exclusive events, then P(A) =
A. 1/4

B. 1/2

C. 3/4

D. 3/8


Answer:

As , S = A ∪ B


∴ P(A ∪ B) = P(S) = 1


As A and B are mutually exclusive events.


∴ P(A ∪ B) = P(A) + P(B)


⇒ 1 = P(A) + 3P(A) [∵ P(A) = 1/3 P(B)]


⇒ 4P(A) = 1


∴ P(A) = 1/4


Our answer matches with option (a)


∴ Option (a) is the only correct choice


Question 32.

One mapping is selected at random from all the mappings of the set A = {1, 2, 3, ….., n) into itself. The probability that the mapping selected is one to one is
A.

B.

C.

D. none of these


Answer:

We know that the no of mapping from a set A to same set containing n elements = nn


As each element have n options to be mapped with n elements.


For mapping selected to be one to one, for first element we have n options ,for second we have n-1 options an so on…


∴ total such mappings = n × (n-1)× (n-2)× … = n!


∴ P(mapping is one to one) =


Our answer matches with option (c)


∴ Option (c) is the only correct choice


Question 33.

If A, B, C are three mutually exclusive and exhaustive events of an experiment such that 3P(A) = 2P(B) = P(C), then P(A) is equal to
A. 1/11

B. 2/11

C. 5/11

D. 6/11


Answer:

Given,


3P(A) = 2P(B) = P(C) = k(say)


∴ P(A) = k/3


P(B) = k/2


And P(C) = k


As events A,B and C are mutually exclusive and exhaustive,


∴ P(A ∪ B ∪ C) = P(A) + P(B) + P(C) = 1


⇒ 1 = k/3 + k/2 + k


⇒ 1 =


⇒ k = 6/11


As, P(A) = k/3 =


Our answer matches with option (b)


∴ Option (b) is the only correct choice


Question 34.

If A and B are mutually exclusive events then
A.

B.

C.

D. none of these


Answer:

As P(A) and P(B) are mutually exclusive event.


∴ P(A ∪ B) = P(A) + P(B)


As P(A ∪ B) ≤ 1


∴ P(A) + P(B) ≤ 1


⇒ P(A) ≤ 1 – P(B)


⇒ P(A) ≤ P(B’)


Our answer matches with option (a)


∴ Option (a) is the only correct choice


Question 35.

If P (A ∩ B) = P (A ∪ B) for any two events A and B, then
A. P (A) = P (B)

B. P (A) > P (B)

C. P (A) < P (B)

D. None of these


Answer:

As, (A ∪ B) = (A ∩ B)


⇒ A and B are same sets.


∴ P(A) = P(B)


Our answer matches with option (a)


∴ Option (a) is the only correct choice


Question 36.

Three numbers are chosen from 1 to 20. The probability that they are not consecutive is
A.

B.

C.

D.


Answer:

3 numbers can be chosen out of 20 in 20C3 ways.


As we have to choose 3 consecutive numbers. So we can’t choose 19 and 18.


Choosing consecutive number is same as choosing a single number because other 2 numbers are automatically chosen.


This can be chosen 18 ways.


∴ P(chosen numbers are consecutive) =


Our answer matches with option (d)


∴ Option (d) is the only correct choice


Question 37.

6 boys and 6 girls sit in a row a row at random. The probability that all the girls sit together is
A.

B.

C.

D. none of these


Answer:

As 6 boys and 6 girls are sitting in a row. So these 12 persons can sit in 12! Ways


Now group all 6 girls together and treat them as 1.


Now, all girls together can sit in 7! Ways


And girls can sit among self in 6! Ways.


∴ total ways in which all 6 girls sit together = 7! × 6!


∴ P(E) =


Hence,


P(E) = 1/132


As our answer matches only with option (d)


∴ Option (d) is the only correct choice.


Question 38.

Without repetition of the numbers, four digit numbers are formed with the numbers 0, 2, 3, 5. The probability of such a number divisible by 5 is
A.

B.

C.

D.


Answer:

Total numbers of 4digit that can be formed using 0,2,3,5 without repetition= 3× 3 × 2 × 1 = 18


For number to be divisible by 5 unit digit must end with 5 or 0.


∴ total such numbers(4 digit) ending with zero are = 3× 2× 1(0 is fixed at the unit digit) = 6


∴ total such numbers(4 digit) ending with zero are = 2× 2× 1(5 is fixed at the unit digit) = 4


Total 4 digit numbers using given digit divisible by 5 are 6 + 4 =10


∴ P(a number divisible by 5) = 10/18 = 5/9


As our answer matches only with option (d)


∴ Option (d) is the only correct choice.


Question 39.

If the probability for A to fail in an examination is 0.2 and that for B is 0.3, then the probability that either A or B fails is
A. > 0.5

B. 0.5

C. ≤ 0.5

D. 0


Answer:

Let A be the event that student ‘A’ fails in exam and B be the event that student ‘B’ fails in exam.


Given, P(A) = 0.2 and P(B) = 0.3


We have to find P(A ∪ B)


We know that P(A ∪ B) = P(A) + P(B) – P(A ∩ B)


As P(A ∩ B) not given, but we know that – 0 ≤ P(A ∩ B) ≤ 1


∴ P(A ∪ B) = 0.2 + 0.3 – P(A ∩ B) = 0.5 - P(A ∩ B)


Clearly a +ve number will be subtracted form 0.5 to make it equal to P(A ∪ B).


∴ P(A ∪ B) ≤ 0.5


As our answer matches with option (c)


∴ Option (c) is the only correct choice.


Question 40.

Three digit numbers are formed using the digits 0, 2, 4, 6, 8. A number is chosen at random out of these numbers what is the probability that this number has the same digits?
A.

B.

C.

D.


Answer:

Total numbers of 3 digit that can be formed using 0,2,4,6,8 = 4× 5 × 5 = 100


3-digit number formed using 0,2,4,6,8 with all digits same are 222,444,666,888


∴ P(number has the same digits) = 4/100 = 1/25


As our answer matches only with option (d)


∴ Option (d) is the only correct choice.